Simulacros de examen

November 9, 2017 | Author: fany1217 | Category: Childbirth, Pregnancy, Knee, Wellness, Health Sciences
Share Embed Donate


Short Description

simulacro CTO...

Description

booksmedicos.org

SIMULACRO 3

1.

Paciente de 48 años de edad que ingresa por dolor abdominal difuso, fiebre, vómitos que últimamente son en posos de café y astenia. A la exploración el abdomen es blando, depresible, sin signos de irritación peritoneal, tensión arterial de 170/100. Analítica: leucocitos 12000/mm3 (72% segmentados, 25% linfocitos, 2% eosinófilos, 1% basófilos), hemoglobina 11g/dl y hematocrito 33%. Los p-ANCA son positivos. En orina presenta proteinuria de 1.5 g/ día, microhematuria. En la radiografía de tórax aparece algún infiltrado pulmonar. Usted sospecha:

1. 2. 3. 4. 5.

2.

Granulomatosis de Wegener. PAN microscópica. Churg-Strauss. Lupus eritematoso sistémico. Vasculitis por hipersensibilidad.

1. Eritema malar. 2. Las manifestaciones sitémicas (astenia, anorexia y pérdida de peso). 3. Proteinuria. 4. Fotosensibilidad. 5. Las alteraciones hematológicas.

4.

Varón de 45 años, con antecedente de hiperuricemia, ingresado en el hospital hace 12 horas por hemorragia digestiva alta. Presenta de forma brusca inflamación de la rodilla derecha con febrícula, siendo evidente en la exploración la presencia de derrame articular. ¿Qué actitud le parece la más correcta?

1. Realizar artrocentesis para confirmar el diagnóstico e iniciar tratamiento con AINE por vía oral. 2. Administrar colchicina por vía oral. 3. Utilizar AINE por vía intramuscular. 4. Combinación de AINE intramusculares y colchicina vía oral. 5. Confirmación de gota por artrocentesis y administración de esteroides intrarticulares.

3.

en estudio por una pancitopenia y proteinuria en rango nefrótico desde hace dos meses. ¿Cuál de las manifestaciones que presenta la paciente NO es un criterio diagnóstico de LES?

Paciente de 32 años que acude a su consulta por la aparición de un exantema eritematoso que se localiza sobre las mejillas y dorso de la nariz respetando el surco nasogeniano y las áreas periorbirarias, que ha empeorado claramente durante la última semana (coincidiendo con unas vacaciones en la playa); astenia y anorexia con pérdida cuantificada de 5 kilogramos en el último mes, y está

Un varón de 70 años con artritis reumatoide de 30 años de evolución consulta por astenia, anorexia, disnea de pequeños esfuerzos, hinchazón en ambas piernas y dolor y tumefacción en rodillas y codos. En la exploración destaca presión venosa yugular elevada, ruidos cardíacos tenues, crepitantes en bases pulmonares y hepatoesplenomegalia. En la analítica destaca: Hb de 10 g/dl, VSG 60, albúmina de 2 g/l y proteinuria de 10 g en 24 horas. ECG con ritmo sinusal a 70 lpm y bajo voltaje. ¿Qué prueba estaría más indicada para establecer el diagnóstico?

1. 2. 3. 4. 5.

5.

Determinación de factor reumatoide. Ecocardiograma. Biopsia renal. Biopsia de grasa subcutánea abdominal. Biopsia de médula ósea.

Varón de 25 años de edad diagnosticado de enfermedad de Crohn que refiere dolor lumbabilateral que empeora con el reposo y rigidez matutina. En la exploración destaca una limitación de la movilidad, así como dolor a la palpación en ambas sacroilíacas y en la radiografía se observa un estrechamiento del espacio articular con esclerosis y algunas erosiones. Respecto a la complicación que presenta este paciente, señale la afirmación CORRECTA:

1. Es muy infrecuente, afectando a menos del 5% de los pacientes.

booksmedicos.org 2. Puede preceder a los síntomas gastrointestinales. 3. Generalmente existen datos radiológicos específicos que permiten la diferenciación de la espondilitis anquilopoyética. 4. A diferencia de la espondilitis anquilopoyética no hay asociación con el HLA-B27. 5. Su evolución está estrechamente ligada al curso clínico de la enfermedad intestinal.

6.

7.

9.

Osteomalacia. Hipervitaminosis D. Osteogénesis imperfecta. Enfermedad de Paget. Síndrome de Albright (displasia fibrosa poliostótica).

Mujer de 65 años de edad que presenta lesiones purpúricas perioculares, pápulas céreas en cuello, axilas e ingles, edema maleolar bilateral que refiere parestesias en 1º, 2º, 3º y mitad radial de 4º dedo bilateralmente. Las parestesias se reproducen al flexionar la muñeca durante un minuto y al percutir sobre la misma. En la analítica destaca proteinuria y creatinina 2 mg/dl. El diagnóstico más probable de esta paciente es:

1. 2. 3. 4. 5.

Una mujer de 35 años consulta por cefalea intensa, disnea de esfuerzo, edemas en ambas piernas y dolor en rodillas y tobillos. Presenta fenómeno de Raynaud. Destaca TA de 190/110, creatinina de 3 mg/dl y hematuria y proteinuria. Los anticuerpos antinucleares tipo antitopoisomerasa 1 (Scl 70), son positivos a títulos altos. ¿Cuál es el diagnóstico más probable?

1. 2. 3. 4. 5.

8.

3. El LCR muestra hallazgos compatibles con meningitis linfocitaria clínica a menudo con elevación de proteínas y cifras de glucosa normales o algo elevadas. 4. La enfermedad se diagnostica reconociendo el cuadro clínico característico y confirmándolo con pruebas serológicas. 5. Un 8% de los pacientes tienen alteraciones cardíacas (bloqueo AV) que como los neurológicos suelen desaparecer.

La existencia en un paciente de sordera, escleras azules, fragilidad ósea, deformidades en extremidades, hipotonía muscular, con transparencia radiológica de los huesos, normocalcemia y normofosforemia, es susceptible de ser diagnosticado de:

1. 2. 3. 4. 5.

SIMULACRO 3

10.

Nefritis lúpica. EMTC. Síndrome CREST. ES con crisis renal. HTA maligna esencial.

Esclerosis sistémica. Fiebre mediterránea familiar. Amiloidosis primaria. Pseudoxantoma elástico Liquen ruber.

Una mujer de 50 años con síndrome de Sjögren de 20 años de evolución, presenta desde hace tres meses fiebre diaria, sudores frecuentes, astenia, anorexia, plenitud postprandial y aparición de bultos en cuello y axilas. En la exploración destaca palidez cutáneo-mucosa, adenopatías laterocervicales bilaterales de dos centímetros de diámetro y hepatoesplenomegalia. En la analítica hay anemia y VSG elevada. En la radiogafía de tórax se observan adenopatías hiliares bilaterales. ¿Qué prueba sería más útil para confirmar el diagnóstico de sospecha?

1. Punción aspiración con aguja fina (PAAF) de una adenopatía cervical. 2. TAC toracoabdominal. 3. Biopsia de médula ósea. 4. Biopsia excisión de una adenopatía cervical. 5. Estudio baritado esófago-gastro-duodenal.

Un anciano vive habitualmente en una finca, con perros entre otros animales. Comenzó con un cuadro de confusión, cefalea intensa, fiebre y rigidez de nuca que es autolimitado y precedido de lesión cutánea característica. Se acompaña de poliartralgias y tenosinovitis generalizada. Señale la opción FALSA: 11. 1. El cuadro corresponde al estadio 2 de un proceso multisistémico causado por espiroquetas y transmitido por garrapatas. 2. La lesión cutánea es característica y se denomina “eritema elevatum diutinum”.

-2-

Un paciente varón de 68 años presenta debilidad muscular de predominio proximal, disfagia y lesiones cutáneas palpebrales eritematosas. Las exploraciones complementarias muestran elevación de CPK, aldolasa, GOT y LDH. En este paciente NO está indicada la realización de:

booksmedicos.org 1. 2. 3. 4. 5.

12.

3. El tratamiento con alopurinol, si el paciente tiene antecedentes de hiperuricemia. 4. La extracción de líquido sinovial para análisis, cultivo y examen de microcristales. 5. Comenzar tratamiento con antibióticos por vía intravenosa.

Búsqueda de neoplasia oculta. Realización de electromiograma. Capilaroscopia. Biopsia muscular. Tratamiento con corticoides.

Una mujer de 63 años, trabajadora de una fábrica textil, consulta a su médico de cabecera porque desde hace unos meses presenta dolor en articulaciones interfalángicas proximales y distales de ambas manos que se acentúa con los movimientos. La exploración revela deformación articular característica. La analítica es normal. Su médico recomienda antiinflamatorios así como jubilación anticipada. Señale el enunciado INCORRECTO al respecto:

15.

1. La capsaicina en crema puede proporcionar alivio sintomático tópico, al interferir con el neuropéptido P. 2. El inhibidor tisular de la metaloproteinasa (TIMP) junto con el inhibidor del activador del plasminógeno 1 (PAI-I) son parte activa en el balance de la destrucción del cartílago. 3. La osteoartritis que padece la paciente descrita es una enfermedad degenerativa sin tratamiento posible salvo sintomático. 4. A diferencia del caso descrito, habitualmente la velocidad de sedimento globular está elevada. 5. Es necesario mantener un cierto grado de ejercicio, preferiblemente isométrico, para asegurar la motilidad articular.

13.

16.

Enfermedad de Buerger. Enfermedad de Wegener. Enfermedad de Eales. Vasculitis primaria del SNC. Enfermedad de Churg-Strauss.

Un niño de 4 años se cortó con unos cristales en la región posterolateral del antebrazo, tras acudir al servicio de urgencias de un centro hospitalario, los médicos descubrieron que estaba incapacitado para extender su dedo pulgar. ¿Qué nervio se encuentra afectado?

1. 2. 3. 4. 5.

Determinación de factor reumatoide. Radiografía simple de tórax. Buscar una paraproteína en plasma. Gammagrafía ósea. Medición de hormona paratiroidea.

17. 14.

Varón de 50 años sin antecedentes de interés que desde hace 4 meses presenta cefalea, pérdida de memoria y en la última semana se ha añadido disminución del nivel de conciencia y varios accidentes isquémicos transitorios en territorios de ambas arterias cerebrales medias. En la analítica destacan solo una VSG elevada, siendo las serologías para sífilis, VIH, CMV, herpes simple, varicela y Borrelia negativas así como los ANA y ANCA. Radiografía de tórax, ecocardiograma y estudio Doppler de troncos supraaórticos sin alteraciones. La TC y la RMN objetivan múltiples infartos en corteza y sustancia blanca. La angiografía no mostró lesiones. En la biopsia cerebral se evidenció afectación de arteriolas con lesiones vasculíticas granulomatosas. El diagnóstico que Ud. sospecha en este paciente es:

1. 2. 3. 4. 5.

Un fumador de 68 años consulta por presentar en los últimos 2 meses dolor intenso en ambas regiones tibiales anteriores, artritis de rodilla y tobillo y acropaquias. Una radiografía muestra periostitis en ambas tibias. ¿Qué estudio complementario, entre los siguientes, estaría indicado realizar, en primer lugar, al paciente?

1. 2. 3. 4. 5.

SIMULACRO 3

Nervio músculo cutáneo. Nervio cubital. Nervio mediano. Nervio radial. Nervio circunflejo.

Un paciente de 40 años, consumidor de cantidades elevadas de bebidas alcohólicas, desarrolla una artritis muy dolorosa de rodilla derecha. ¿Cuál de las siguientes sería la actuación urgente más adecuada?

Un joven de 18 años fue herido con un arma blanca en la cara anterior del codo, con lo cual perdió la capacidad de flexionar las articulaciones interfalángicas proximales de todos los dedos. ¿Qué nervio se encuentra afectado?

1. La obtención de radiografías de ambas rodillas. 2. El análisis de los niveles de ácido úrico en sangre.

1. Nervio cubital. 2. Nervio mediano.

-3-

booksmedicos.org 3. Nervio radial. 4. Nervio circunflejo. 5. Nervio músculo cutáneo

18.

Un varón de 17 años presenta episodios de edema de partes blandas en miembros superiores e inferiores, con poca respuesta a corticoides. Además, en una extracción dentaria presentó un intenso edema de cara y cuello que respondió mal a la adrenalina. El estudio alérgico es negativo. En el estudio del complemento se encuentra un C4 bajo, con déficit de C1 inhibidor. Posiblemente se trate de:

1. 2. 3. 4. 5.

19.

Un paciente sano, varón de 32 años, que es sometido a una intervención sin incidencias de microdiscectomía lumbar L5-S1 6 semanas antes, acude a consulta de forma prematura porque ha comenzado con un dolor en aumento y de características severas en la zona lumbar que le despierta por la noche. El examen físico revela una herida de características normales siendo la exploración neurológica compatible con la normalidad. Los estudios de laboratorio muestran una VSG de 90 mm/h y unos leucocitos en niveles de 9000/mm3. La radiografía de la zona resulta normal. ¿Cuál considera que es el siguiente paso más adecuado en el manejo de este paciente?

1. Antibióticos orales que cubran Stafilococo aureus. 2. Repetir los estudios de laboratorio en una semana y reevaluar la VSG. 3. RNM con gadolinio. 4. Biopsia del espacio discal intervenido. 5. Irrigación y desbridamiento de la herida quirúgica en quirófano.

Artritis reumatoide. Arteritis de la arteria temporal. Edema angioneurótico familiar. Ataxia telangiectasia. Amiloidosis.

Varón de 56 años que acude a su médico por dolor abdominal y plenitud postpandrial. Ha perdido peso últimamente y comenta que no se encuentra bien. En la exploración llama la atención unas lesiones papilomatosas, hiperqueratósicas, de color grisáceo en axilas e ingles. ¿De qué cuadro se trata?

1. 2. 3. 4. 5.

20.

21.

SIMULACRO 3

22.

Condilomas acuminados. Queratosis seborreica. Acantosis nigricans. Papilomatosis gigante. Epidermodisplasia verruciforme.

Paciente de 25 años de edad que acude a Urgencias tras sufrir accidente esquiando. Refiere focalidad traumática en su rodilla derecha. A la exploración presenta dolor en la cara interna de la rodilla que se exacerba al forzar el valgo a 30 grados de flexión. Las pruebas meniscales son negativas. En lo que respecta a la patología que sufre el paciente, señale la respuesta correcta:

1. Requiere una RMN para su diagnóstico. 2. Es muy probable que tenga derrame articular. 3. La exploración con la rodilla en extensión completa tiene idéntico significado. 4. Su lesión es la más frecuente de las lesiones de los ligamentos de la rodilla. 5. Cuando se identifica una verdadera lesión aislada es necesario tratamiento quirúrgico.

Acude a su consulta una joven de 20 años con una lesión hiperpigmentada en la espalda, de 5 cm. de diámetro, más estrecha por un extremo que por otro y bordes irregulares. Dice que toda la vida ha tenido un “lunar” ahí, pero que no era tan grande, aunque tampoco lo había observado mucho dada su localización. ¿Cuál sería su actitud ante esta paciente?

23.

1. Biopsia excisional de la lesión completa, para realizar un correcto estudio anatomopatológico. 2. Biopsia incisional por ser una lesión demasiado grande para extirparla sin diagnóstico previo. 3. Curetaje de la lesión estudiando anatomopatológicamente la muestra obtenida. 4. Excisión quirúrgica hasta la fascia con márgenes de seguridad de 1 cm. 5. Seguimiento regular de la lesión, que casi con seguridad es un caso de nevus displásico.

Mujer de 76 años que es llevada al servicio de urgencias tras sufrir caída casual presentando focalidad traumática en cadera derecha. Como antecedentes presenta cardiopatía isquémica e insuficiencia renal crónica. A la exploración destaca el importante acortamiento y rotación externa de la extremidad. En unos días aparece un gran hematoma sobre la zona de la cadera. El tratamiento más adecuado para este paciente consiste en:

1. Abstinencia quirúrgica por el riesgo anestésico. 2. Prótesis parcial cementada.

-4-

booksmedicos.org 3. Osteosíntesis con enclavado trocantérico. 4. Tracción esquelética durante 3 semanas y posteriormente apoyo progresivo. 5. Osteosíntesis con tornillos canulados cervicales.

3. Uveítis incipiente. 4. Queratitis herpética. 5. Queratitis por amebas.

27. 24.

Paciente de 20 años de edad que acude tras recibir un puñetazo en su ojo derecho. El paciente afirma que desde que recibió el golpe ve doble. La AV es 1 en ambos ojos. Las pupilas son isocóricas y normorreactivas. En el polo anterior sólo presenta un leve hiposfagma. El fondo de ojo es normal. A la palpación se aprecia la existencia de crepitación y la exploración de la motilidad ocular extrínseca pone de manifiesto una limitación de la elevación del ojo derecho. En esta situación su diagnóstico de sospecha es:

1. 2. 3. 4. 5.

Fractura de suelo orbitario. Fractura de pared medial de la órbita. Fractura de techo orbitario. Fractura de pared temporal de la órbita. Simulación.

Mujer de 78 años de edad. Acude al hospital por dolor localizado en el canto interno de su ojo derecho de 3 días de evolución. Desde hace muchos años le llora mucho ese ojo. Se le propuso cirugía, pero la paciente decidió no operarse porque le daba mucho miedo. De entre las siguientes opciones terapéuticas señale la que considere más acertada:

1. Programar para dacriocistorrinectomía al día siguiente. 2. Tratamiento antibiótico sistémico, y unas semanas después llevar a cabo dacriocistorrinostomía. 3. Dacriocistectomía. 4. Tratamiento antibiótico sistémico y dacriocistectomía. 5. Tan sólo tratamiento antibiótico tópico.

26.

Paciente varón de 28 años de edad, con infección VIH de largo tiempo de evolución. Abandonó el tratamiento hace varios años y está ingresado en un estadio de inmunosupresión muy avanzada. Le llaman porque el paciente refiere pérdida brusca de visión en su ojo izquierdo en los últimos días. A la exploración presenta una retina con abundantes hemorragias y exudados. Con respecto al diagnóstico más probable, señale la correcta:

1. 2. 3. 4. 5.

28.

25.

SIMULACRO 3

Paciente de 22 años, natural de Colombia, que acude a la consulta por rinorrea purulenta y bloqueo nasal bilateral progresivo de 4 meses de evolución. En las últimas semanas aparecen hundimiento del dorso nasal y varios episodios de epistaxis. La rinofibroscopia demuestra una ulceración septal grande con bordes mucosos necróticos. Se solicita TC de senos paranasales en donde no aparecen otros hallazgos además de la perforación septal. Ante la sospecha de un proceso tumoral se toma biopsia de la lesión que es informada como infiltrado inflamatorio polimorfo de predominio linfoide pequeño, sin granulomas ni células gigantes y con inmunohistoquímica positiva para CD2, CD56 y CD3 citoplasmático. ¿Cuál es el tratamiento de elección de la patología que presenta la paciente?

1. 2. 3. 4. 5.

Paciente que refiere desde hace 3 días sensación de cuerpo extraño, legañas y secreciones en ambos ojos. Ha notado además la aparición de un bulto justo delante de su oreja derecha. Desde el punto de vista exploratorio, la AV es 1 en ambos ojos y presenta una inyección mixta intensa, así como folículos en la conjuntiva tarsal inferior. Señale cuál es el diagnóstico más probable:

29.

1. Conjuntivitis alérgica. 2. Conjuntivitis vírica.

-5-

Leucemia intrarretiniana. Tumor de Kaposi coroideo. Retinitis por CMV. Coroiditis por toxoplasma. Obstrucción de vena central de la retina.

Ciclofosfamida y esteroides. Quimiorradioterapia. Ciclo antibiótico prolongado con ciprofloxacino. Anfotericina B y cirugía. Cirugía nasosinusal y radioterapia.

Mujer de 30 años con antecedente de otitis en la infancia y expuesta a un importante estrés laboral que presenta desde hace 10 días otalgia derecha intensa, sin otorrea, que no mejora con tratamiento antibiótico tópico y sistémico. A la exploración presenta una perforación timpánica central en oído derecho sin supuración y dolor a la palpación en región preauricular derecha. El diagnóstico de sospecha será:

booksmedicos.org 1. Reagudización de otitis media crónica simple. 2. Reagudización de otitis media crónica colesteatomatosa. 3. Otitis media aguda resistente al tratamiento. 4. Otitis externa maligna. 5. Disfunción de la articulación temporomandibular.

30.

33.

Mujer de 36 años, tercigesta, de 35 semanas de amenorrea. En la exploración física llama la atención una altura uterina menor a la que corresponde. En la ecografía se ve estancamiento del crecimiento de los diámetros abdominales respecto a la eco anterior, con diámetro biparietal y longitud femoral normales. El eco-doppler fetal muestra aumento de resistencia al flujo en arterias placentarias. El registro cardiotocográfico basal es no reactivo. ¿Cuál sería la siguiente actuación para valorar el estado fetal?

34.

32.

Paciente de 42 años con carcinoma de cuello, diagnosticado por biopsia que invade el labio anterior y 1 cm de la cúpula vaginal; los parametrios no están invadidos; la rectoscopia, la urografía descendente y la cistoscopia son normales. En este caso, el tratamiento más correcto sería:

1. Radioterapia externa más quimioterapia. 2. Radioterapia de contacto más quimioterapia. 3. Histerectomía total ampliada tipo WertheimMeigs más radioterapia externa. 4. Radioterapia de contacto seguida de histerectomía total simple. 5. Ninguna de las anteriores.

La curva de temperatura basal en una mujer de 28 años muestra 36,5º C a 36,6º C durante todo un ciclo de 32 días. Debe considerarse que es un ciclo:

1. 2. 3. 4. 5.

Las vacunas del papilomavirus frente al carcinoma cervical han demostrado su eficacia mediante:

1. La demostración de que no aparece cáncer cervical en los vacunados. 2. La demostración de que se producen anticuerpos tras la vacunación. 3. La demostración de protección frente a la aparición de lesiones preneoplásicas (CIN 2/3) asociadas a los virus incluidos en la vacuna. 4. La demostración de la remisión del cáncer cervical en las mujeres vacunadas. 5. Estudios clínicos no controlados.

1. Amnioscopia. 2. Registro cardiotocográfico estresante. 3. Determinación de HCG, alfa feto-proteína y estriol no conjugado. 4. Inducción del parto. 5. Amniocentesis tardía.

31.

SIMULACRO 3

35.

Anormal por ser excesivamente largo. Totalmente normal. Anormal por posible anovulación. Normal pero con ovulación algo atrasada. Anormal por niveles de testosterona altos.

Una mujer de 50 años, menopáusica hace 2 años, presenta metrorragias. En una biopsia de endometrio se diagnostica hiperplasia endometrial adenomatosa y atípica de grado moderado. Sus únicos datos anormales son una glucemia basal de 120 mg/dl y obesidad leve. ¿Cuál es el tratamiento más indicado?

1. Histerectomía. 2. Progestágenos orales cíclicos durante 6 meses (tipo acetato de medroxiprogesterona). 3. Vigilancia y repetición de la biopsia cada 6 meses. 4. Anticonceptivos orales de dosis baja de estrógeno durante 6 meses. 5. Análogos de GnRH 6 meses.

Primigesta de 32 años que acude a urgencias refiriendo sangrado genital y prueba de embarazo en farmacia positiva. Se realiza exploración ginecológica y los hallazgos ecográficos y analíticos sugieren el diagnóstico de mola vesicular. ¿Cuál sería el tratamiento de elección?

1. Quimioterapia con metrotexate. 2. Legrado uterino con legra de Recamier y pinza Winter. 3. Legrado por aspiración. 4. Histerectomía simple (conservando ovarios). 5. Histerectomía radical.

36.

-6-

Una mujer de 65 años consulta por metrorragias escasas desde hace tres meses. La exploración ginecológica es normal. En una ecografía transvaginal se observa un útero de 7 x 3 x 4 cm con un endometrio de 14 mm de espesor, y unos ovarios atróficos. La citología cérvicovaginal informa de un frotis atrófico sin otras alteraciones celulares.

booksmedicos.org En una biopsia endometrial realizada por aspiración con una cánula flexible de tipo Cornier o Pipelle (microlegrado) se informa de escaso material endometrial de tipo atrófico, insuficiente para un diagnóstico endometrial adecuado. ¿Cuál de las siguientes indicaciones es la más adecuada?

1. 2. 3. 4. 5.

1. Tratamiento hemostático con estrógenos + progestágenos, seguido de progestágenos cíclicos cada mes, durante 6 meses. 2. Histerectomía. 3. Repetición de la biopsia si vuelve a sangrar, y en caso contrario, repetición de la ecografía a los 4-6 meses. 4. Histeroscopia. 5. Completar el estudio con determinación de marcador tumoral CA 12,5 y otras pruebas de imagen com RNM o TAC.

37.

41.

No necesita más tratamiento. Radioterapia abdominal. Poliquimioterapia adyuvante. Radioterapia más quimioterapia con cisplatino. Cirugía de rescate si recidiva.

42. 1. 2. 3. 4. 5.

39.

Quiste. Displasia fibrosa. Fibroadenoma. Cáncer. Hamartoma.

Síndrome del ovario poliquístico. Fallo ovárico autoinmune. Prolactinoma hipofisario. Sinequias uterinas (síndrome de Asherman). Tumor cerebral.

Mujer de 52 años, con amenorrea desde hace 4 meses, que consulta por la presencia de artromialgias, cefaleas, sofocos e irritabilidad en los últimos meses. Entre sus antecedentes destaca el ser fumadora de un paquete de tabaco al día y el ser hipertensa en tratamiento. Entre las recomendaciones que usted le haría a esta paciente NO se encuentra:

1. 2. 3. 4. 5.

Mujer de 25 años con nódulo mamario palpable de aparición brusca. La ecografía revela un nódulo anecogénico, de límites muy precisos, morfología regular y refuerzo posterior, único, de 3 cm de diámetro. ¿Cuál de los que se relacionan es el diagnóstico más probable?

Fibroadenoma. Quiste. Displasia mamaria. Carcinoma. Mamografía normal para la edad de la paciente.

Una paciente de 30 años que consulta por amenorrea secundaria presenta unas concentraciones plasmáticas basales de: FSH 2 mU/ml, LH 1,5 mU/ ml, prolactina 9 ng/ml. Tras la administración de acetato de medroxiprogesterona, 10 mg por día durante 5 días, no se observa sangrado vaginal. En cambio, tras la administración de estrógenos equinos conjugados, 1,25 mg por día durante 25 días, aparece una menstruación. De las siguientes causas de amenorrea, ¿cuál es la que más se corresponde con el cuadro clínico?

1. 2. 3. 4. 5.

Mujer de 68 años que comienza con clínica de aumento del diámetro abdominal desde hace 5 meses, que se acompaña de dolor en los últimos 15 días. Tras realizar las exploraciones pertinentes se le diagnostica de cáncer de ovario de tipo cistoadenocarcinoma seroso, en estadio IIIb. Se le realiza tratamiento quirúrgico sin dejar ningún resto tumoral, ¿cómo completaría el tratamiento?

1. 2. 3. 4. 5.

38.

40.

SIMULACRO 3

Disminuir el consumo de sal. Abandonar el tabaco. Tratamiento hormonal sustitutivo. Ejercicio físico moderado. Aumentar los productos lácteos de la dieta.

Una mujer gestante en su 23 semana presenta dolor lumbar derecho persistente, y en la ecografía se objetiva únicamente dilatación pieloureteral derecha moderada. No tiene fiebre y el sedimento urinario es normal. La actitud a seguir debe ser:

1. Efectuar una urografía i.v. de control cada dos semanas hasta el parto. 2. Tratar con analgésicos y efectuar control ecográfico periódico. 3. Administrar antibióticos para evitar la infección. 4. Provocar el parto para prevenir complicaciones ulteriores. 5. Desaconsejar nuevos embarazos.

Paciente de 38 años con nódulo mamario indoloro, de bordes imprecisos. La mamografía revela imagen nodular, con espículas en todos sus márgenes, y 10 microcalcificaciones finas, agrupadas en el interior. El diagnóstico más probable, entre los que se citan, es:

-7-

booksmedicos.org 43.

Gestante de 37 semanas a la que se le practica monitorización fetal no estresante con el siguiente resultado: frecuencia cardíaca fetal 135 lpm, ondulatoria normal, movimientos fetales escasos, sin ascensos en la frecuencia cardíaca y sin deceleraciones. ¿Qué actitud tomaría?

1. 2. 3. 4. 5.

1. Permitir la evolución espontánea evitando la ayuda manual durante el periodo expulsivo. 2. Dejar evolucionar hasta periodo expulsivo y practicar maniobras de ayuda manual para abreviar el periodo expulsivo tras la salida del ángulo inferior de la escápula. 3. Permitir la evolución hasta periodo expulsivo y finalizar éste mediante aplicación de fórceps para evitar la distocia de cabeza última. 4. Terminar el parto mediante cesárea. 5. Determinar la actitud de la cabeza fetal mediante una radiografía simple del abdomen materno.

Cesárea urgente. Inducción del parto. Amnioscopia. Amniocentesis tardía. Prueba de Pose.

47. 44.

Una primigesta de 21 años, con amenorrea de 12 semanas, prueba de embarazo positiva y cuadro de emesis gravídica acude a consulta por presentar metrorragia moderada. En la exploración se encuentra un útero mayor que amenorrea, ausencia de latido fetal y ecografía característica de mola vesicular. La radiografía P-A de tórax no evidencia metástasis pulmonares. El tratamiento indicado es evacuación uterina, que deberá ir seguido de:

1. 2. 3. 4. 5.

45.

46.

Embarazada en el primer trimestre de gestación, con antecedentes de hijos fallecidos por una inmunodeficiencia y cuyo defecto molecular en la familia ya es conocido. ¿Qué prueba diagnóstica es la más idónea para el diagnóstico prenatal de la inmunodeficiencia?

1. 2. 3. 4. 5.

Controles semanales del título de beta-HCG. Histerectomía. Monoquimioterapia. Poliquimioterapia. Radioterapia.

48.

Gestante de 32 semanas con sangrado vaginal oscuro y escaso. A la palpación abdominal se aprecia una marcada hipertonía uterina. No se aprecia latido fetal. El estado materno es malo, con signos de shock y analítica sanguínea de Urgencias compatible con el inicio de una coagulopatía. ¿Qué diagnóstico le sugeriría?

1. 2. 3. 4. 5.

SIMULACRO 3

Una paciente sometida a cesárea por inducción fallida tras amniorrexis prematura presenta en su tercer día de puerperio malestar general, fiebre de 38,5º C, loquios fétidos, útero subinvolucionado y doloroso a la palpación. ¿Cuál es el diagnóstico más probable?

1. 2. 3. 4. 5.

Placenta previa sangrante. Abdomen agudo de causa no obstétrica. Desprendimiento prematuro de placenta. Amenaza de parto pretérmino. Tumoración sangrante del canal del parto.

49.

Una paciente secundigesta, con antecedente de parto vaginal anterior, ingresa en su 34ª semana de gestación por parto en curso. Mediante tacto vaginal se diagnostica dilatación de 6 cm y presentación pelviana (nalgas puras), variedad S.I.I.A., en II plano. Ecográficamente se estima que el peso fetal estará comprendido entre 2.100 y 2.300 g. Señale la conducta obstétrica correcta:

Estudio molecular en los padres. Biopsia de corion en la 9ª semana. Determinación de alfa-fetoproteía en la madre. Amniocentesis. Estudio de subpoblaciones de linfocitos en la madre.

Endometritis puerperal. Dehiscencia de la histerectomía. Retención de restos placentarios. Necrosis isquémica de mioma uterino. Absceso de pared abdominal.

Mujer de 24 años, primigesta, que en una revisión en la 24ª semana de gestación presenta cifras tensionales de 140/85 (cifras en la primera consulta de 120/65) que se repiten en la siguiente consulta. En las últimas dos semanas ha aumentado 2 kg de peso. En la analítica de orina hay proteinuria de +++. No presenta ninguna otra sintomatología. ¿Cuál de las siguientes actuaciones NO sería correcta en esta situación?

1. Tratamiento hipotensor con metildopa. 2. Reposo en decúbito lateral izquierdo.

-8-

booksmedicos.org 3. Dieta con suficiente aporte de proteínas. 4. Control de la tensión arterial 4 veces al día. 5. Control diario de peso.

50.

1. En este caso, la aparición de defectos campimétricos sería excepcional. 2. Los microprolactinomas son los tumores hipofisarios funcionantes más frecuentes. 3. Es muy poco probable que presente déficit hormonales hipofisarios. 4. Sería recomendable iniciar tratamiento con dosis bajas de cabergolina. 5. Los fármacos son la causa más frecuente de hiperprolactinemia.

Mujer de 27 años, delgada, asintomática, remitida ante el hallazgo de una glucosuria importante. Como antecedentes familiares destaca padre y dos hermanos con diabetes en buen control en tratamiento con antidiabéticos orales. En la consulta le realiza una glucemia capilar en ayunas con cifras de glucosa de 116 mg/dl. ¿Cuál es la respuesta FALSA?

53.

1. La glucosuria asintomática es el primer síntoma. 2. Asocian sordera neurógena. 3. Estaría indicado hacer una sobrecarga oral de glucosa. 4. Tiene una herencia autosómica dominante. 5. Es una mutación en el factor nuclear hepático 1-α.

51.

Varón de 73 años, que acude al servicio de Urgencias estuporoso. En la exploración destaca fiebre, deshidratación mucocutánea importante, con signo del pliegue positivo. Analíticamente: glucemia venosa: 870 mg/dl, osmolaridad plasmática 320 mOsm/kg, Na 155 mEq/l, K 3,5 mEq/l, pH 7,30 con bicarbonato normal y cuerpos cetónicos negativos. ¿Cuál es falsa?

Paciente varón de 43 años presenta cifras de colesterol total de 310 mg/dl con cifras de triglicéridos de 134 mg/dl. Hace tres años presentó un episodio de angina. Además presenta HTA en tratamiento con un betabloqueante y un calcioantagonista y el año pasado le dijeron que tenía cifras de glucemia algo elevadas, aunque niega ser diabético. Refiere que en su familia hay varias personas con tratamiento hipolipemiante pero desconoce las cifras de colesterol y el tipo de tratamiento. A la exploración únicamente destaca sobrepeso grado 2. ¿Cuál es el diagnóstico de mayor sospecha?

1. 2. 3. 4. 5.

1. Es necesario descartar la presencia de un foco infeccioso desencadenante del cuadro. 2. La hidratación del paciente es la medida más importante, aunque también es recomendable el tratamiento con insulina en perfusión continua para disminuir la hiperglucemia y la diuresis osmótica. 3. Este cuadro tiene una elevada mortalidad, entre 5-20%. 4. Pueden asociarse manifestaciones neurológicas como convulsiones o hemiplejia transitoria. 5. El cuadro no justifica que el paciente esté estuporoso, por lo que de forma inmediata deberemos solicitar una TC craneal.

52.

SIMULACRO 3

54.

Mujer de 58 años, que acude a consultas de endocrinología para valoración de hiperprolactinemia. En la analítica destaca la existencia de prolactina de 105 ng/ml, y al solicitar RMN de silla turca se visualiza una imagen hipodensa de 4 mm en la adenohipófisis, sugestivo de microadenoma. Clínicamente se encuentra asintomática, y presenta amenorrea desde hace 8 años. Señale la opción FALSA:

Hipercolesterolemia familiar poligénica. Disbetalipoproteinemia familiar. Hipercolesterolemia familiar monogénica. Hipercolesterolemia familiar combinada. Ninguna de las anteriores.

Paciente mujer de 41 años consulta por episodios de ansiedad, sudoración fría, palidez, palpitaciones, con frecuencia a primera hora de la mañana, desapareciendo tras el desayuno. Entre sus antecedentes personales destacan HTA, en tratamiento farmacológico, múltiples cólicos nefríticos de repetición y colecistectomía por colelitiasis. En la analítica presenta: Glu: 63 mg/dl, creatinina: 0.79 mg/dl; urea: 42 mg/dl, Na: 138 mEq, K: 3.7 mEq/l, calcio: 11 mg/dl, fósforo: 3.2 mg/dl, PTH elevada. Entre los antecedentes familiares nos cuenta que su madre falleció siendo joven, de un tumor intestinal que no sabe precisar, habiendo presentado múltiples problemas digestivos. Señale la afirmación FALSA en relación con el síndrome de mayor sospecha:

1. Tiene una herencia autosómica dominante. 2. Con frecuencia se asocia a tumores hipofisarios. 3. Está relacionada con el gen de la menina, presente en el cromosoma 11. 4. Hasta en un 95% los pacientes presentan ganglioneuromatosis. 5. La forma de presentación de la paciente no es la más frecuente.

-9-

booksmedicos.org 55.

Paciente de 85 años institucionalizado en una residencia. Presenta disfagia secundaria a enfermedad de Alzheimer, por lo que es portador de sonda nasogástrica para nutrición enteral. Como tratamiento habitual tiene pautado furosemida, digoxina y lactulosa. Desde hace 5 días en tratamiento antibiótico con ciprofloxacino por presentar infección urinaria. Comienza con cuadro de diarrea de más de 6 deposiciones líquidas al día. Señale, de las siguientes opciones lo que usted haría:

ha mantenido determinaciones de tiroglobulina y Ac antitiroglobulina negativos. En la revisión actual, la paciente presenta una determinación de tiroglobulina negativa pero la determinación de Ac antitiroglobulina se ha hecho positiva. ¿Cuál sería su actitud ante estos hallazgos?

1. Continuaría con el seguimiento como hasta ahora ya que la tiroglobulina sigue siendo negativa. 2. Aumentaría la dosis de L-tiroxina ya que el resultado nos indica que la dosis es insuficiente. 3. Bajaría la dosis de L-tiroxina ya que el resultado nos indica que la dosis es excesiva. 4. Realizaría una TC cérvico-torácica con contraste para valorar posibles adenopatías. 5. Solicitaría una ecografía cervical para descartar recidiva local accesible a reintervención y repetiría el tratamiento con I-131.

1. Deberíamos suspender la lactulosa y parar la nutrición enteral, dejando al paciente únicamente con sueroterapia durante unos días hasta que desaparezca la diarrea. 2. Deberíamos tratarla con enlentecedores del tránsito, como loperamida o codeína. 3. Si la diarrea persiste más de 36-48 horas estaría indicado realizar coprocultivo y toxina de Clostridium difficile para descartar una infección intestinal. 4. Si la administración de la nutrición era continua deberíamos pasarla a bolos. 5. Convendría pasar a un preparado con fibra no fermentable.

56.

58.

Paciente de 36 años que consulta por TSH: < 0,01 mcUI/ml en el contexto de clínica catarral y dolor cervical anterior irradiado al oído derecho. La paciente refiere pérdida de peso en los últimos 3 meses acompañada de palpitaciones y temblor. A la exploración cervical se objetiva un tiroides ligeramente aumentado de tamaño, con lóbulo tiroideo derecho mayor que el izquierdo y sin evidente dolor. ¿Cuál sería su actitud ante esta paciente?

Acude a su consulta una niña de 2 años y medio por pérdida de peso de varios meses de evolución. La madre refiere que la niña se encuentra irritable e inapetente. Las deposiciones son pastosas y amarillentas, en torno a 5-6 al día. Entre sus antecedentes destacan dos ingresos hospitalarios, uno a los 6 meses por bronquiolitis y otro al año y medio por broncoespasmo. Fue alimentada toda su lactancia con fórmula artificial y la diversificación alimentaria fue supervisada por su pediatra. En la exploración física nos encontramos una leve distensión abdominal, una talla en el percentil 50 y un peso en el percentil 3. En una analítica básica hecha por su pediatra destaca una hemoglobina de 9,8 g/dl con un VCM de 70 fl. Ante esta paciente, ¿qué actitud le parece más adecuada?

1. Colonoscopia y analítica de sangre y heces para descartar organicidad. Si todo fuera normal, tranquilizaría a los padres y les informaría de la benignidad del proceso. 2. Determinación de IgA antitransglutaminasa, test del sudor y estudio bioquímico de heces. Si los anticuerpos son positivos, indicaría una dieta exenta de gluten hasta realizar una biopsia duodenal. 3. Determinación de IgA antitransglutaminasa, de inmunoglobulinas totales y test del sudor. Si serología positiva, endoscopia para obtener biopsia duodenal. 4. Determinación de IgA antitransglutaminasa, inmunoglobulinas totales y estudio de HLA. Si el estudio genético es positivo, indicaría una dieta libre de gluten. 5. Incidir en la anamnesis acerca de la introducción de la alimentación, pruebas alérgicas para las proteínas de la leche de vaca y dieta exenta de lactosa.

1. La diagnosticaría de enfermedad de Graves-Basedow e instauraría tratamiento con antitiroideos. 2. La diagnosticaría de tiroiditis subaguda, pautaría betabloqueantes y antiinflamatorios y le indicaría que es un proceso autolimitado que se resolverá espontáneamente. 3. Solicitaría nueva analítica con TSH, T4L y una gammagrafía tiroidea. 4. Solicitaría nueva analítica con TSH, T4L y ecografía cervical. 5. Solicitaría nueva analítica con TSH, T4L y PAAF del lóbulo tiroideo derecho.

57.

SIMULACRO 3

Paciente de 34 años de edad que acude para revisión por antecedente de carcinoma papilar de tiroides hace 4 años, tratado mediante tiroidectomía total y tratamiento ablativo con 100 mCi de I-131 posteriormente. Desde entonces la paciente

-10-

booksmedicos.org 59.

Paciente de 2 años que acude por fiebre alta de 4 días de evolución, acompañada de tos, hiperemia faríngea, conjuntivitis bilateral exudativa y un exantema máculo-papular confluente que comenzó por cara y afecta a palmas y plantas. Respecto a este cuadro, ¿cuál es la etiología y su orientación terapéutica?

controlado, presenta temblores generalizados a las 4 horas de vida. En la analítica se objetiva una glucemia de 20 mg/dl, un sodio de 139 mEq/l, potasio 4 mEq/l, calcio total 7,8 mg/dl, 25500 leucocitos/ mm3 y una hemoglobina de 21 g/dl con un hematocrito del 68%. Respecto a la actitud a seguir en este paciente señale lo incorrecto:

1. Streptococcus pyogenes y amoxicilina-clavulánico oral 7 días. 2. Desconocida y gammaglobulina intravenosa en los primeros 10 días de la enfermedad. 3. Virus del grupo Togaviridae y aciclovir intravenoso 5 días. 4. Virus del grupo Paramyxoviridae y tratamiento sintomático. 5. Virus herpes humano tipo 6 y tratamiento sintomático.

60.

61.

1. La hipoglucemia neonatal, sea o no sintomática, debe ser tratada por riesgo de secuelas neurológicas. 2. La gran leucocitosis y la hipoglucemia del paciente obliga a hacer un chequeo completo de infección, especialmente al tratarse de una gestación mal controlada. 3. Se considera policitemia patológica del recién nacido a cifras de hematocrito que sobrepasen el 65%. 4. La madre probablemente no se realizó controles analíticos durante la última parte de la gestación. 5. Este recién nacido muy probablemente tenga un riesgo mayor del habitual de padecer alguna alteración cardiológica.

Recién nacido, varón de 15 días de edad, que presenta ictericia y acolia, con escasa ganancia ponderal. Presenta hepatomegalia dura a la palpación abdominal, siendo el resto de la exploración normal. Se le practica una biopsia hepática y se recibe el siguiente informe anatomopatológico: fibrosis portal, proliferación ductal y trombos biliares intraportales. ¿Cuál es el diagnóstico más probable?

1. 2. 3. 4. 5.

63.

Síndrome de Alagille. Hepatitis neonatal idiopática. Atresia de vias biliares extrahepáticas. Hepatitis congénita por citomegalovirus. Enfermedad de Gilbert.

Respecto a la prevención de la transmisión vertical madre-hijo de la infección por VIH es incorrecto decir:

1. La lactancia materna se contraindica en nuestro medio. 2. El riesgo de transmisión perinatal se incrementa a partir de 12 horas de rotura de membranas. 3. El mayor número de infecciones connatales por VIH se adquieren en el parto. 4. El recién nacido ha de recibir tratamiento antirretroviral que se debe iniciar antes de las 6 horas de vida y mantener durante al menos 4 semanas. 5. Las técnicas serológicas no son fiables en el diagnóstico de infección VIH en el niño durante los 18 primeros meses.

Lactante de 2 meses de edad de origen asiático cuyos padres traen al servicio de urgencias por aparición de coloración amarillenta progresiva desde el nacimiento. El embarazo fue normal, con parto a las 41 semanas eutócico y en cefálica, con peso al nacimiento de 3.020 kg. Alimentada con lactancia materna exclusiva con buena ganancia ponderal. A la exploración presenta ictericia verdínica generalizada, abdomen globuloso, hepatomegalia de 2 cm y polo de bazo. Usted indaga sobre el aspecto de las heces y los padres comentan que son más claras de lo normal, en ocasiones blanquecinas. Se realiza bioquímica sanguínea que objetiva los siguientes valores: bilirrubina total 28,1 mg/dl, bilirrubina directa 5,9 mg/dl, GOT 338 U/l, GPT 170 U/L, GGT 233 U/l, fosfatasa alcalina 350 U/l y LDH 675 U/l. Respecto a la patología que usted sospecha, señale la afirmación falsa:

1. Se trata de una ictericia colestática. 2. El tratamiento definitivo de esta entidad es quirúrgico. 3. Con frecuencia se observa dilatación del colédoco en la ecografía abdominal con vesícula biliar dilatada. 4. Existe un trastorno en la excreción de bilis. 5. El diagnóstico se confirma con biopsia hepática.

64. 62.

SIMULACRO 3

Un recién nacido de 36 semanas de gestación y 3900 gramos de peso, fruto de un embarazo poco

-11-

Respecto a la infección urinaria en la infancia, señale la opción incorrecta de entre las siguientes:

booksmedicos.org 1. Es una de las infecciones más frecuentes en la infancia después de la infección de vías respiratorias y la gastroenteritis. 2. La vía hematógena es más frecuente en los lactantes pequeños. 3. Es más frecuente en el sexo femenino, excepto en neonatos en que es igual de frecuente en ambos sexos. 4. Los gérmenes más frecuentemente implicados son gramnegativos. 5. En varones con fimosis es típica la infección por Proteus mirabilis.

65.

espirometría que muestra una FVC de 3450 ml (82%), FEV1 de 1840 ml (52%) y FEV1/FVC 0,53, con prueba broncodilatadora negativa. La analítica general, la Rx tórax y el ECG son normales. ¿Cuál sería el planteamiento terapéutico inicial?

1. Abandono de tabaco, salbutamol a demanda y tiotropio pautado. 2. Abandono de tabaco, salbutamol a demanda y fluticasona pautada. 3. Abandono de tabaco, salbutamol a demanda e indacaterol pautado. 4. Abandono de tabaco, salbutamol a demanda y tiotropio pautado junto con formoterol pautado. 5. Abandono de tabaco, pauta corta de corticoides orales y asociación pautada de salmeterol y fluticasona.

Lactante varón de 7 meses de vida que acude a urgencias por presentar restos hemáticos mezclados con las heces en las 5 últimas deposiciones, sin cambios en la consistencia ni la frecuencia de las mismas. A la exploración el paciente está activo y vital, bien hidratado y perfundido y presenta una temperatura de 36,3º C con frecuencia cardíaca de 107 lpm y tensión arterial de 105/65. Usted aprecia leve palidez facial, la auscultación cardiopulmonar es normal, el abdomen es blando y depresible, sin masas ni megalias. En el tacto rectal sólo se aprecian restos hemáticos y a la inspección anal no se observa nada anormal. Decide dejarlo en observación, donde el paciente mantiene buen estado general y hace una toma de lactancia artificial sin incidencias, pero una hora después de su llegada a urgencias presenta de nuevo otro episodio de rectorragia. Respecto a la patología que sospecha, señale la afirmación correcta:

67.

Paciente de 74 años con carcinoma de células grandes de 4 cm de diámetro en LSD, sin adenopatías mediastínicas patológicas en la TAC y sin captaciones patológicas a ningún otro nivel en la PET. La exploración funcional respiratoria muestra un FEV1 de 2450 ml (81%) y una DLCO de 82%. Señale cuál es la actitud más adecuada:

1. Realización de test de esfuerzo cardiopulmonar para medir el consumo máximo de oxígeno. 2. Realización de mediastinoscopia. 3. Realización de lobectomía superior derecha. 4. Realización de TAC o RNM cerebral. 5. Realización de gammagrafía ósea.

1. Probablemente se trate de una diarrea invasiva por lo que recogería coprocultivo y dejaría ingresado para observación y sueroterapia intravenosa. 2. Probablemente se trate de una invaginación intestinal por lo que solicitaría una ecografía abdominal urgente y llevaría a cabo una reducción hidrostática si se confirma el diagnóstico. 3. Probablemente se trate de una alergia a las proteínas de la leche de vaca por lo que recomendaría sustituir la fórmula habitual por hidrolizado de proteínas. 4. Probablemente se trate de una fisura anal por lo que recomendaría una pomada antiinflamatoria. 5. Probablemente de trate de una malformación congénita del tubo digestivo cuyo tratamiento es quirúrgico.

66.

SIMULACRO 3

68.

Paciente de 54 años de edad, hipertenso con mal control a pesar de la utilización de tres fármacos antihipertensivos, que es derivado a su consulta por ser roncador habitual. El paciente no es obeso, y no refiere hipersomnolencia diurna ni ningún otro síntoma. Le realiza usted una polisomnografía con los siguientes resultados: IAR 21, SatO2 media durante el sueño 97%, SatO2 mínima durante el sueño 92%. Cuál le parece que sería la actitud correcta con este paciente:

1. Tiene un síndrome de apnea del sueño, y se debe considerar el tratamiento con CPAP nocturno. 2. No tiene un síndrome de apnea del sueño, es un roncador simple. 3. Tiene un síndrome de apnea del sueño, pero dado que está asintomático, no está justificado en ningún caso el tratamiento con CPAP. 4. Tiene un síndrome de obesidad-hipoventilación. Se debe iniciar tratamiento con ventilación mecánica no invasiva nocturna.

Un paciente de 69 años de edad, fumador activo con IPA acumulado de 45, consulta por tos y expectoración crónicas de años de evolución, a lo que en los últimos 10 meses se ha unido disnea de grandes esfuerzos, lentamente progresiva hasta hacerse de moderados esfuerzos. Le realiza usted una

-12-

booksmedicos.org 5. Tiene un síndrome de hipoventilación alveolar primaria. Se debe iniciar tratamiento con ventilación mecánica no invasiva.

5. Realización de radiografía de tórax, y si es normal alta con cita para test de mesa basculante.

72. 69.

Paciente de 61 años de edad sometido hace año y medio a un trasplante de pulmón bilateral por EPOC. Hace correctamente el tratamiento inmunosupresor pautado. Acude a su consulta refiriendo un cuadro muy lentamente progresivo de tos seca y ligera disnea de grandes esfuerzos. La exploración física y la radiografía de tórax son normales. Señale cuál de las siguientes pruebas le pondrá, con más probabilidad, sobre la pista diagnóstica:

1. 2. 3. 4. 5.

70.

Espirometría. Broncoscopia con biopsia transbronquial. Cultivo de esputo. Serología a gérmenes atípico. Investigación de micobacterias en esputo.

73.

Una de las siguientes constituye una contraindicación para la realización de valvuloplastia mitral percutánea:

1. 2. 3. 4. 5.

74.

71.

Un varón hipertenso de 78 años presenta cansancio y debilidad en ambas piernas al caminar que se alivia con el reposo. También refiere alteraciones en la erección. ¿Qué patología presenta el paciente?

1. Obliteración femoropoplítea. 2. Obliteración ilíaca izquierda. 3. Estenosis del canal vertebral lumbar (pseudoclaudicación). 4. Síndrome de Leriche. 5. Obliteración ilíaca derecha.

Un paciente de 63 años de edad presentó un infarto de miocardio anterior hace un mes. Se le ha realizado un ecocardiograma, que ha mostrado una fracción de eyección del 40% y un cateterismo cardíaco, que ha mostrado una enfermedad de un vaso (DA media). Se requiere hacer un estudio de viabilidad miocárdica. Para ello, la prueba más útil será:

1. 2. 3. 4. 5.

SIMULACRO 3

Ecocardiograma-dobutamina. Ergometría. Talio-dipiridamol. Resonancia cardíaca. Ecocardiograma-dipiridamol.

Disfunción ventricular izquierda. Fibrilación auricular. Insuficiencia aórtica severa. Insuficiencia mitral severa. Antecedentes de embolismo sistémico.

Varón de 26 años que consulta por episodios de palpitaciones ocasionales mal toleradas. La EF es normal, así como el ECG basal. En un Holter de 24 h presenta extrasístoles ventriculares muy frecuentes con origen en el tracto de salida de VD. ¿Cuál es la actitud más adecuada?

1. Iniciar tratamiento betabloqueante. 2. Iniciar tratamiento calcioantagonista. 3. Descartar cardiopatía estructural mediante ecocardiograma y/o resonancia magnética cardiaca. 4. Iniciar tratamiento con amiodarona. 5. Seguimiento clínico.

Un paciente con infarto crónico anteroseptal hace 6 años, se presenta consciente y asintomático en el centro de salud por haber presentado una pérdida brusca de conocimiento con recuperación espontánea en unos segundos sin otros síntomas. El electrocardiograma realizado muestra una onda Q en las derivaciones V1-4, ritmo sinusal a 70 latidos por minuto con PR normal. La actitud más razonable a llevar a cabo será:

75.

1. Administración de tratamiento antianginoso, antiagregación y alta a su domicilio. 2. Derivación a un centro hospitalario. 3. Alta, recomendando vuelva a consultar si recidiva la clínica. 4. Realización de gasometría arterial y bioquímica, y si es normal alta.

En el paciente de la pregunta anterior, las pruebas de imagen son normales. Se inició tratamiento betabloqueante a dosis bajas. En el Holter posterior, las extrasístoles seguían siendo frecuentes y el paciente refiere astenia e impotencia. ¿Cuál es la actitud más adecuada?

1. Cambiar de betabloqueante. 2. Iniciar calcioantagonistas. 3. EEF y ablación de la arritmia

-13-

booksmedicos.org 4. Iniciar amiodarona. 5. Como no tiene cardiopatía estructural, retirar betabloqueantes y seguimiento sin tratamiento.

76.

ba puesto el cinturón de seguridad. A la atención inicial la paciente presenta GCS 6, sin asimetría a la exploración motora y PICNR. Se realiza intubación de la paciente y traslado a centro hospitalario para ser valorada. Tras la estabilización inicial se realiza TC craneal en el que no se observan colecciones hemáticas ni fracturas, si bien se observa un punteado hemorrágico a nivel del esplenio del cuerpo calloso. En relación con la patología que presenta la paciente NO es cierto que:

Paciente de 72 años portador de derivación ventrículo peritoneal colocada por una hidrocefalia normotensa hace 2 años con buena respuesta al tratamiento, de manera que el paciente ha podido volver a realizar vida independiente. Desde hace 2 meses el paciente se queja de cefalea opresiva, sobre todo cuando se incorpora de la cama, en algún momento acompañado de náuseas y vómitos. Además, la familia lo encuentra con mayor lentitud de pensamiento, dificultad para la capacidad de abstracción y problemas de memoria; además presenta problemas para la marcha que se confirman a la exploración mostrando imposibilidad para levantar contra gravedad el miembro inferior izquierdo. No refiere fiebre. Se ha realizado TC craneal que no muestra sangrados ni aumento del tamaño ventricular, si bien se observan dos colecciones hipodensas en espacio subdural de ambas convexidades. Lo más probable que le ocurre a este paciente es:

1. Parece corresponder a una lesión axonal difusa. 2. En este caso la lesión es de grado III. 3. Estaría indicada la colocación de monitor de sensor presión intracraneal para valorar la necesidad de aplicar medidas para la reducir la misma. 4. Implican la desconexión axonal neuronal por mecanismos de aceleración-desacaleración. 5. Son lesiones asociadas a gran morbi-mortalidad.

79.

1. Obstrucción drenaje ventricular. 2. Progresión de su cuadro de hidrocefalia normotensa. 3. Infección, siendo el germen más probable S. epidermidis. 4. Hiperfunción del drenaje ventricular. 5. Asociación a demencia tipo Alzheimer.

77.

Varón de 64 años diagnosticado hace 2 años de adenocarcinoma de pulmón tratada mediante cirugía y quimioterapia posterior. El paciente ha seguido revisiones en consulta de oncología sin encontrar progresión de la enfermedad. Hace 15 días comienza con una hemianopsia derecha sin otra focalidad neurológica por lo que se realiza estudio de imagen que observa la presencia de dos lesiones occipitales izquierdas con captación de contraste en anillo y edema asociado. Se repite TC-body que no evidencia lesiones a otros niveles. El tratamiento más indicado en el momento actual sería:

Un paciente de 64 años, hipertenso y diabético, con antecedentes de linfoma tratado con varios ciclos de quimioterapia, actualmente en remisión, consulta por diplopia binocular multidireccional, horizontal y vertical. A la exploración se observa limitación de todos los movimientos del ojo derecho salvo abducción, ptosis palpebral derecha y midriasis arreactiva derecha. Cuando se le pide mirar abajo y a la izquierda se observa ausencia de incliclotorsión del ojo derecho. Además de los hallazgos oculomotores solo se pone de manifiesto hipoestesia supraciliar y maxilar derecha. Respecto al cuadro que presenta el paciente es correcto:

1. Dados los antecedentes de diabetes sospecha una mononeuritis isquémica del tercer par derecho. 2. El paciente presenta afectación del III y IV par izquierdo así como de V2, por lo que dados sus antecedentes solicitaría una RM para descartar infiltración a nivel de la fisura orbitaria superior derecha. 3. El paciente presenta afectación del III y IV par izquierdo así como de V2, por lo que dados sus antecedentes solicitaría una RM para descartar infiltración a nivel del seno cavernoso derecho. 4. El paciente presenta una lesión del III par de causa compresiva, por lo que habría que descartar aneurisma de la comunicante posterior derecha como primera posibilidad. 5. La asociación de síntomas del paciente sitúa la lesión a nivel pontino derecho.

1. 2. 3. 4.

Radioterapia holocraneal. Control de su enfermedad primaria. Radiocirugía. Cirugía de resección de las lesiones y radioterapia holocraneal. 5. Corticoides y antiepilépticos.

80. 78.

SIMULACRO 3

Mujer de 45 años que sufre accidente de tráfico con salida de la carretera y vuelco del vehículo. Lleva-

-14-

Un varón de 71 años, diabético insulinodependiente, consulta por pérdida de fuerza en miembros superiores. El paciente se queja de difultad para

booksmedicos.org la manipulación fina de objetos con las manos. El trastorno comenzó hace 2 meses afectando a una mano y progresivamente se extendió a la otra. A la exploración, el paciente muestra debilidad moderada en ambas manos con áreas de clara atrofia en ambas eminencias. Destaca cierto piramidalismo de la marcha y leve disartria. Los reflejos musculares están exaltados en ambas piernas y brazo izquierdo. El reflejo mentoniano es vivo. Durante la exploración se observan fasciculaciones en raíz de los miembros y lengua. Respecto al trastorno que padece señale la correcta:

2. Si sus hijos expresan más de 40 repeticiones del triplete CAG en el cromosoma 4 desarrollarán con alta probabilidad el trastorno si viven lo suficiente. 3. Está indicado realizar una prueba de neuroimagen para comprobar la imagen típica de realce mesencefálico. 4. El tratamiento es exclusivamente sintomático pero no altera el curso evolutivo. 5. El que sea él quien trasmita la enfermedad supone que si sus hijos heredan la mutación desarrollarán antes y de forma más severa el trastorno que si lo hiciera la madre.

1. Se trata de una enfermedad de la placa neuromuscular. Solicita EMG y anticuerpos anti-receptor de acetilcolina. 2. La distribución de la debilidad corresponde con una polineuropatía diabética. Solicita EMG y refiere al endocrino para mejorar control. 3. La mayoría de las veces la enfermedad que padece tiene agregación familiar por una mutación en la superóxido dismutasa. 4. En su evolución es raro encontrar afectación oculomotora y esfinteriana. 5. Nunca se produce afectación cognitiva.

81.

83.

Benzodiazepinas a dosis de relajante muscular. Anticomiciales del perfil de la carbamazepina. Baclofeno. Tizanidina. Infiltración de toxina botulínica. 84.

82.

Varón de 56 años, fumador e hipertenso, que consulta por dolor lumbar de varias meses de evolución. El dolor irradia por cara posterolateral de miembro inferior derecho hasta el la cara dorsal del pie y primer dedo. A pesar de la toma regular de analgésicos, AINE y periodos cortos de relajantes musculares, el dolor no cede aunque todavía es capaz de realizar su trabajo habitual. Cuál de las siguientes opciones respecto a la patología que presenta el paciente no es correcta:

1. El dolor probablemente empeorará con la flexión del tronco y las maniobras de Valsalva, mejorando con la extensión del tronco y el decúbito supino. 2. En la exploración podemos encontrar dificultad en la marcha taloneante, un Lasegue directo positivo a 30 en miembro inferior derecho y los reflejos osteotendinosos conservados. 3. La RM mostrará una hernia discal L5-S1 posterolateral derecha y el EMG datos de denervación aguda en el territorio de la raíz S1 derecha. 4. Antes de optar por la cirugía, indicaría tratamiento rehabilitador. 5. El tratamiento quirúrgico consistiría en hemilaminectomía, flavectomía y discectomía.

Una paciente de 45 años consulta por la aparición de contracciones involuntarias en la muñeca izquierda, que se flexiona contra su voluntad quedando en una contracción mantenida y dolorosa. El fenómeno desaparece con el sueño. El trastorno la incapacita considerablemente. El resto de la exploración es normal. En el manejo de la paciente la opción terapéutica más adecuada es:

1. 2. 3. 4. 5.

SIMULACRO 3

Un paciente de 42 años comienza con movimientos involuntarios que afectan a lengua y manos. Son de carácter incesante, irregulares y arrítmicos. No sigue ni ha seguido ningún tratamiento farmacológico habitual. La exploración es normal pero durante la entrevista muestra cierta irritabilidad. La familia refiere que notan un cambio en su personalidad. Interrogado por su historia familiar refiere que su madre murió de un infarto de miocardio y su padre se suicidó a los 60 años. Señale la falsa:

1. La enfermedad que presenta constituye la forma más frecuente de corea hereditario.

-15-

Varón de 48 años sin antecedentes de interés que acude por dolor cervical irradiado hacia la cara lateral del antebrazo y 1er y 2º dedo de la mano izquierda. Además refiere que no puede coger las bolsas de la compra con el brazo izquierdo y que se ha caído en dos ocasiones porque le fallan las piernas. No refiere antecedentes de interés. En la exploración física la maniobra de Spurling es positiva, la fuerza es normal salvo en la flexión del brazo y extensión de la muñeca izquierda (4/5). El reflejo bicipital está abolido, si bien los patelares y aquíleos están exaltados con respuesta cutáneo plantar extensora bilateral. Respecto a la patología que presenta el paciente señale la respuesta incorrecta.

booksmedicos.org 1. Lo más probable es que presente una compresión radicular de la raíz C6 izquierda y una mielopatía compresiva, secundario todo ello a una voluminosa hernia discal cervical C5-C6. 2. Uno de los diagnósticos diferenciales que deberíamos tener en cuenta es la esclerosis lateral amiotrófica. 3. Puede existir un déficit sensitivo en el dermatomo correspondiente a la raíz C6 izquierda e incluso en miembros inferiores. 4. La prueba de elección para el diagnóstico correcto es una tomografía computarizada con inyección intratecal de gadolinio. 5. La cirugía consiste en realizar una discectomía por vía anterior, dejando un injerto intersomático en el espacio discal.

85.

86.

1. Intubación orotraqueal, eco-doppler transcraneal urgente y si presenta velocidades arteriales elevadas, iniciar tratamiento con vasodilatadores intravenosos. 2. Realización de RM urgente para descartar isquemia de tronco del encéfalo y, en caso de confirmarse la isquemia, realizar fibrinolisis. 3. Solicitar TC craneal, si no existe hidrocefalia ni resangrado iniciar tratamiento con aporte de volumen y, si es necesario, drogas vasoactivas para aumentar la tensión arterial. 4. Realizar arteriografía urgente para inyectar papaverina intraarterial y así mejorar la autorregulación cerebral. 5. Mantener en reposo al paciente y aumentar la dosis de nimodipino.

Mujer de 26 años que acude a urgencias por episodio de diplopía intermitente de instauración súbita y horas de evolución. Entre los antecedentes destaca un episodio de parestesias en miembro inferior derecho hace seis meses, del que se recuperó completamente de semanas. La exploración física muestra un defecto en la aducción del ojo derecho, que se acompaña de un nistagmo del ojo izquierdo a la levoversión. Teniendo en cuenta el diagnóstico más probable de esta paciente, ¿qué prueba diagnóstica sería más útil para confirmar el diagnóstico?

1. 2. 3. 4. 5.

SIMULACRO 3

87.

Velocidad de conducción nerviosa. Electromiograma. TAC craneal con contraste. Resonancia magnética. TAC torácico.

Acude a urgencias un varón de 73 años de edad, hipertenso y diabético con buen control, que tras levantarse y durante el desayuno presenta atragantamiento no pudiendo digerir el mismo. Se acompaña de sensación brusca de nauseas, y visión doble. Al incorporarse de la silla, es incapaz de mantener la bipedestación y tiene que sentarse. A la llegada a urgencias se encuentra consciente, con TA de 173/85 mmHg, glucemia de 127 mg/dl, y afebril. En la exploración física se objetiva hipoestesia de miembros izquierdos y hemicara derecha, ptosis de ojo derecho y miosis derecha. Una radiografía de tórax es normal. ECG demuestra ritmo sinusal. Entre las siguientes afirmaciones del cuadro clínico que está sufriendo esté paciente señale la respuesta correcta:

1. La arteria más probablemente afectada es la arteria cerebelosa posteroinferior izquierda. 2. Dado que presenta hipoestesia en hemicara derecha, con toda probabilidad, la lesión asentará en la hemiprotuberancia derecha. 3. Dado los factores de riesgo del paciente y la presencia de ritmo sinusal, lo más probable es que se deba a un cardioembolismo. 4. Una disección de la arteria vertebral derecha puede ser causa de este cuadro. 5. Tras la realización de TAC craneal, si usted no encuentra hemorragia cerebral, comenzará de forma inmediata con anticoagulación.

Varón de 57 años, fumador, ingresado en la unidad de cuidados intensivos por bajo nivel de conciencia. En la exploración neurológica presentaba GCS 8 (escala WFNS 4), pupilas isocóricas y nornorreactivas. La TC realizada mostró una hemorragia que se distribuye por las cisternas basales, ambas cisuras de Silvio y cisura interhemisférica, sin hemorragia intraventricular, sin hidrocefalia ni hematoma intraparenquimatoso (Fisher III). Es intubado, sedado y relajado y se inicia tratamiento con nimodipino por sonda nasogástrica. Se realiza arteriografía que muestra aneurisma de arteria comunicante anterior que se emboliza. Seis días después se retira la sedación y el paciente recupera el nivel de conciencia, alcanzando un GCS de 14, sin embargo, presenta de forma brusca hemiparesia derecha y afasia, con una TA de 98/60 y una saturación con gafas nasales del 95%. Señale cuál es la actitud correcta ante la situación del paciente.

88.

-16-

Mujer de 35 años, si antecedentes de interés, que es valorada en el Servicio de Urgencias tras presentar una crisis parcial simple motora, en forma de movimientos tónicos en el miembro inferior izquierdo. A la anamnesis dirigida refiere una cefalea progresiva en la semana previa, que la despierta por las noches y aumenta con las maniobras de Valsalva. También presenta una clínica de más de

booksmedicos.org un mes de evolución consistente en otalgia y otorrea derechas. En relación con el cuadro que probablemente presente la paciente señale la opción INCORRECTA:

usted solicita un estudio coproparasitológico en heces en el que se visualizan estructuras quísticas mediante la tinción de Kinyoun. De las siguientes, señale la combinación CORRECTA entre el microorganismo causante del cuadro y su tratamiento de elección:

1. La TC craneal mostrará una lesión hipodensa en el lóbulo temporal derecho con captación periférica de contraste “en anillo”. 2. La etiología más probable incluye estreptococos del grupo viridans y anaerobios. 3. La presencia de fiebre y signos de irritación meníngea es frecuente. 4. El estudio microbiológico del LCR presenta escasa rentabilidad. 5. El tratamiento empírico se basa en una cefalosporina y metronidazol.

1. Isospora belli-Mebendazol. 2. Mycobacterium bovis-Isoniacida, pirazinamida, rifampicina y etambutol. 3. Salmonella enteritidis-Ciprofloxacino. 4. Cryptosporidium parvum-Cotrimoxazol. 5. Enterocytozoon bieneusi-Fumagilina.

91.

89.

Un varón de 43 años con el diagnóstico de leucemia mieloide aguda tipo M6 de la clasificación FAB recibe tratamiento de inducción con citarabina e idarrubicina. Es portador de un catéter venoso central tunelizado permanente. Al cabo de 15 días del primer ciclo el hemograma muestra una neutropenia profunda (50 neutrófilos/mcl), con discreta anemia (hemoglobina 12 g/dl) y trombopenia (99.000 plaquetas/mcl). Tres días después comienza con tos no productiva y fiebre elevada. Tras extraer hemocultivos, se inicia tratamiento empírico con meropenem y daptomicina. La radiografía de tórax es normal, y el paciente persiste febril al cabo de 72 horas de tratamiento. El servicio de Microbiología informa de que los hemocultivos son estériles. En ese momento, usted solicita una determinación de galactomanano sérico, que resulta positiva. Señale cuál de los siguientes fármacos asociaría en ese momento al tratamiento:

1. 2. 3. 4. 5.

90.

SIMULACRO 3

Un paciente suyo, diagnosticado de púrpura trombótica idiopática refractaria, ha sido recientemente sometido a una esplenectomía terapéutica. Le pregunta cuáles son los riesgos que implica su nueva condición de esplenectomizado. ¿Cuál de los siguientes no incluiría usted en su respuesta?

1. 2. 3. 4. 5.

92.

¿Cuál es el medio de cultivo específico para Legionella spp.?

1. 2. 3. 4. 5.

Anfotericina B liposomal. Factores estimulantes de colonias (GM-CSF). Fluconazol. Anfotericina B deoxicolato. Voriconazol.

93.

Un varón de 45 años, que fue diagnosticado de infección por VIH-1 hace cinco años, decidió abandonar tanto el seguimiento como el tratamiento antirretroviral durante todo ese periodo. Acude ahora a su consulta refiriendo un cuadro de diarrea acuosa, muy copiosa (hasta doce deposiciones diarias), sin productos patológicos, de aproximadamente dos meses de evolución. A la exploración física el paciente aparece caquéctico, con lesiones sobreelevadas y blanquecinas en los bordes de la lengua que no se despegan al raspado. Entre otras exploraciones complementarias,

Agar chocolate. Agar Mac Conkey. Medio TSI (triple sugar iron). Agar BCYE-alfa. Medio NNN (Novy, MacNeal, Nicolle).

Dos varones de 15 años, tras una excursión campestre por la vera de un río, desarrollan un cuadro de fiebre, tos y disnea progresiva. En la radiografía de tórax se aprecia un patrón intersticial bilateral. Los dos pacientes refieren el haber estado “jugando” con unos topillos muertos que encontraron en su camino. ¿Cuál le parece la etiología más probable de la infección pulmonar de estos dos pacientes?

1. 2. 3. 4. 5.

-17-

Enfermedad tromboembólica venosa. Sepsis y meningitis tras la mordedura de un perro. Sepsis meningocócica fulminante. Infección grave por Bordetella holmesii. Neumonía por Rhodococcus equi.

Fiebre botonosa mediterránea. Fiebre Q. Peste. Histoplasmosis pulmonar. Tularemia.

booksmedicos.org 94.

1. 2. 3. 4. 5.

Mujer de 67 años que es diagnosticada de endocarditis sobre válvula mitral protésica metálica producida por Enterococcus faecalis. En el ecocardiograma se observó una vegetación de 8 mm de longitud mayor en la cara auricular de la válvula. No se observó disfunción valvular. La paciente fue tratada inicialmente con la combinación de ampicilina y gentamicina y quedó afebril. Los primeros hemocultivos de control fueron negativos. Tras 4 semanas de tratamiento la paciente presenta de nuevo fiebre y en los hemocultivos se aísla de nuevo E. faecalis con el mismo patrón de sensibilidad. ¿Qué actitud le parece más adecuada en este momento?

97.

98.

¿Cuál de las siguientes pautas de triple terapia frente a VIH se pueden administrar en una sola dosis diaria y mediante la toma de un solo comprimido:

1. 2. 3. 4. 5.

¿Cuál es el tratamiento de elección de un quiste hidatídico localizado en el hígado que mide 11 cm de diámetro mayor?

Diez días después de haber presentado un cuadro de diarrea, un niño de 11 años desarrolla un cuadro febril sin una focalidad infecciosa evidente. En la analítica destaca una creatinina de 2,3 mg/dl, unos niveles de plaquetas de 56.000 por microlitro y un nivel de hemoglobina de 8,2 g/l. El volumen corpuscular medio de los hematíes es de 88. El paciente presenta un perfil hepático normal salvo un nivel de bilirrubina de 2,3 y un aumento de la LDH sérica (875). Se realizó una extensión de sangre periférica en la que se observaron algunos esquistocitos. ¿Cuál cree usted que fue el resultado del coprocultivo?

1. 2. 3. 4. 5.

AZT + Abacavir + Lopinavir/ritonavir. Abacavir + Tenofovir + Efavirenz. Emtricitabina + Tenofovir + Raltegravir. Emtricitabina + Tenofovir + Darunavir / ritonavir. Emtricitabina + Tenofovir + Efavirenz. 99.

96.

Azitromicina. Doxiciclina. Levofloxacino. Quinupristina - dalfopristina. Amoxicilina - ácido clavulánico.

1. Actitud expectante. 2. Tratamiento médico con albendazol. 3. Punción, aspiración del contenido y posterior inyección de una sustancia escolicida como el suero hipertónico. 4. Resección quirúrgica. 5. Cualquiera de las anteriores sería correcta.

1. En estos casos lo más probable es la endoftalmitis que precisará de enucleación. 2. Lo más probable es que la evolución de la endocarditis esté siendo favorable. Lo indicado es descartar un foco alternativo de infección (por ejemplo urinaria) para justificar el cuadro actual de bacteriemia. 3. Dado el fracaso del tratamiento antibiótico se debe sustituir ampicilina por vancomicina. 4. Se debería proceder a tratamiento quirúrgico cardiaco para sustitución de la válvula afecta. 5. Se debería adoptar una actitud expectante dado que es la evolución natural de una endocarditis enterocócica. Lo habitual es que los hemocultivos sean positivos durante los primeros 3 meses tras el inicio del tratamiento.

95.

SIMULACRO 3

Varios miembros de la misma familia son atendidos en un hospital por el desarrollo de un cuadro de fiebre, artralgias y mialgias, cefalea, astenia intensa y tos sin expectoración. En sus radiografías de tórax se observa un patrón intersticial bilateral. Las analíticas demuestran una elevación moderada de enzimas hepáticos. Interrogados sobre una posible fuente de exposición común a todos ellos, refieren que habían estado viendo un partido de fútbol en la habitación donde aquella noche había parido una gata. Señale cuál le parece el tratamiento más adecuado para los integrantes de esta familia:

Flora saprofita. Se aisló Escherichia coli O157:H7. Se aisló Campylobacter jejuni. Se aisló Clostridium difficile. Se aisló Entamoeba histolytica.

Señale la respuesta correcta, respecto de la infección del ser humano por el virus de la gripe H1N1:

1. Se ha demostrado que el inicio del tratamiento con oseltamivir en las primeras 48 horas del cuadro clínico mejora el pronóstico de pacientes con neumonía por virus de la gripe que precisan ingreso hospitalario. 2. Oseltamivir, al igual que zanamivir, son fármacos cuyo mecanismo de acción es la inhibición de la hemaglutinina de la envoltura del virus de la gripe. 3. Se puede, y se debe, administrar la vacuna antigripal a los sujetos inmunodeprimidos, dado que este tipo de vacuna se fabrica con virus inactivados.

-18-

booksmedicos.org

bio, que las proteínas sean superiores a 150 mg/ dl, que la glucosa esté tan disminuida y que las células que predominen en el LCR no sean los monocitos. 5. Si el paciente estuviera vacunado frente a neumococo disminuiría la posibilidad de meningitis por esta bacteria.

4. Los cambios que sufre el virus de la gripe obligan a la vacunación anual de todos aquellos sujetos en los que esta vacuna esté indicada. 5. El personal sanitario tiene la responsabilidad de vacunarse anualmente frente a la Gripe para intentar disminuir el riesgo de transmitir la infección a los pacientes que atienda. Esta medida es especialmente relevante si atiende habitualmente a pacientes inmunodeprimidos. 102. 100.

¿Qué antibiótico emplearía para tratar una infección producida por Escherichia coli productor de betalactamasa de espectro extendido - BLEE-?

1. 2. 3. 4. 5.

¿Qué fármaco se puede emplear para tratar por vía oral la infección por Leishmania?

1. 2. 3. 4. 5.

Amoxicilina. Amoxicilina-ácido clavulánico. Cefepime. Piperacilina-tazobactam. Meropenem. 103.

101.

SIMULACRO 3

Varón de 79 años, en tratamiento con esteroides desde hace varios meses por una arteritis de la temporal. Consulta por un cuadro de doce horas de evolución consistente en fiebre de hasta 39,5º C y dolor cervical. Su familia le encuentra con cierta “obnubilación” y un lenguaje incoherente. A la exploración física destaca la rigidez de nuca notable que presenta el paciente. Se realiza una TC craneal que descarta signos de neuroimagen sugestivos de hipertensión intracraneal. Se realiza una punción lumbar en la que se obtiene un líquido turbio con las siguientes características citobioquímicas: células 1.200 por microlitro (90% PMN, 10% M); proteínas 4.500 mg/dl; glucosa 24 mg/dl. La determinación simultánea de glucosa en sangre fue de 175 mg/dl. Señale la respuesta que considere FALSA:

Glucantime o algún otro antimonial pentavalente. Anfotericina B liposomal. Alopurinol. Miltefosina. Ninguno de los anteriores.

Paciente varón de 45 años de edad que acude al médico de atención primaria por aparición de edemas maleolares. Entre sus AP destacan obesidad de larga evolución, actualmente con IMC 45, HTA y DMII ambas de reciente diagnóstico y con deficiente control. Se realiza analítica de sangre y orina: Hb 12,5 g/dl, leucocitos 8600 (fórmula normal), plaquetas 245000, Glu 132 mg/dl, Cr 1,6 mg/ dl, urea 62 mg/dl, Na 143 meq/l, K 4,6 meq/l, Alb 3 g/l, PT 6,5. orina: destaca proteinuria +++, 1-2 hematíes/campo. Es remitido a nefrología donde se realiza estudio inmunológico con resultado normal y se cuantifica proteinuria, siendo de 4 g/día. Se realiza biopsia renal. ¿Qué resultado espera encontrar y qué tratamiento considera más adecuado? 1. GN Mesangiocapilar de tipo I  Prednisona + Ciclofosfamida. 2. GN Membranosa  Tratamiento antiproteinúrico con IECA/ARA II. 3. GN Focal y Segmentaria  Tratamiento antiproteinúrico con IECA/ARA II + Tratamiento intensivo para perder peso. 4. GN Membranosa  Tratamiento antiproteinúrico con IECA/ARA II + Tratamiento inmunosupresor, ya que presenta deterioro de función renal. 5. Hallazgos de Nefropatía diabética  Tratamiento antiproteinúrico con IECA/ARA II.

1. La edad del paciente y el tratamiento con esteroides son factores de riesgo para meningitis tuberculosa, pero la instauración brusca del cuadro clínico y la preeminencia de PMN en el LCR son datos que van en contra de este diagnóstico. 2. La meningitis por Listeria monocytogenes es más frecuente en mayores de 50 años y en aquellos con inmunosupresión celular, como los tratados con esteroides. Pero el predominio de PMN en el LCR imposibilita este diagnóstico, ya que en la meningitis por Listeria predominan los monocitos. 3. Por la edad del paciente podría tratarse de una meningitis por Streptococcus pneumoniae, además es lo que corresponde a las características citobioquímicas del LCR de este paciente. 4. Es poco probable la meningitis por virus ya que en este caso es excepcional que el líquido sea tur-

104.

-19-

Paciente mujer de 58 años de edad con AP de hepatopatía por VHC que ingresa por desarrollar en pocos días edemas y lesiones purpúricas palpables en ambos MMII. Refiere descenso del ritmo de diuresis en las últimas 48 horas. En la analítica de ingreso destaca Hb 9,9 g/dl, Hcto 29%, leuco-

booksmedicos.org citos 5500 (FN), plaquetas 44.000, Glu 92mg/dl, urea 182 mg/dl, Cr 4,2 mg/dl, Na 129 meq/l, K 5,6 meq/l. En orina destaca proteinuria de 3,6 g/día no selectiva y hematuria microscópica. Respecto al proceso que sufre la paciente es cierto todo excepto:

No toma tratamiento alguno. En los días previos al ingreso bradipsiquia y somnolencia progresivas con dificultad para realizar tareas sencillas. Además refiere astenia y pérdida de 5 kilos de peso a lo largo de los últimos tres meses. En la analítica de ingreso presenta Hb 11,4 g/dl, Hcto 33%, plaquetas 286000, leucocitos 7450 (FN), Glu 96 mg/dl, urea 32 mg/dl, Cr 0,8 mg/dl, Na 121 meq/l, K 4,0 meq/l, Cl 101 meq/l, Osm 298 mosm/l. Orina: Ph 5,5, Na 35 meq/l, Osm 130 meq/l, sin alteraciones del sedimento. ¿Qué prueba diagnóstica realizaría a continuación?

1. Es probable que a lo largo del ingreso la paciente precise tratamiento renal sustitutivo. 2. La lesión histológica subyacente en el riñón puede ser una GN Mesangiocapilar tipo I. 3. La lesión histológica subyacente puede ser una GNRP de tipo III. 4. Es probable que a lo largo del ingreso la paciente precise tratamiento con plasmaféresis. 5. Al realizar el estudio inmunológico encontraremos hipocomplementemia con descenso de C3 y C4.

105.

1. 2. 3. 4. 5.

Paciente varón de 66 años de edad con AP de HTA, sobrepeso, DM II y EPOC grave que ingresa en UCI por sepsis de origen urinario. Precisa IOT. En la Rx de tórax se observa dudoso foco de consolidación con sospecha de neumonía aspirativa. Lo más probable es que presente:

108.

1. Acidosis metabólica con anión gap alto y alcalosis respiratoria. 2. Acidosis metabólica con anión gap normal, sin compensación respiratoria. 3. Acidosis mixta con anión gap normal. 4. Acidosis metabólica, parcialmemente compensada con hiato aniónico normal. 5. Acidosis mixta con anión gap alto.

106.

TAC craneal. Ecografia abdominal. Rx de torax. Electroencefalograma. No precisa más pruebas diagnósticas.

Paciente mujer de 32 años de edad con AP de trastorno bipolar en tratamiento crónico con litio. Ingresa en psiquiatría con síntomas de fase maniaca aunque refiere buen cumplimiento terapéutico. Durante el ingreso se cursa interconsulta a nefrología por poliuria de 7-8 litros al día con polidipsia y nicturia. Analítica de sangre sin alteraciones. Osmolaridad en orina 140 mosm/l. La sospecha diagnóstica y la actitud más adecuada será:

1. Ingesta subrepticia de diuréticos. Reinterrogar y registrar los cajones de la mesilla de noche. 2. Diabetes insípida por litio. No precisa pruebas diagnósticas de confirmación, se debe suspender cuanto antes tratamiento con litio. 3. Potomanía. No precisa otras pruebas diagnósticas ni actitud terapeútica alguna. 4. Probable tubulopatía por ingesta crónica de litio. Está indicado realizar un test de deshidratación. 5. Tubulopatía proximal por litio. Suspender tratamiento con litio tras descartar potomanía con un test de deshidratación.

Paciente mujer de 26 años con ERC secundaria a GN focal y segmentaria. En analítica de control de metabolismo óseo mineral destaca: Ca 8 mg/dl, P 6 mg/dl, PTH 550 pg/ml (normal de 16-65 pg/ml, en pacientes en diálisis hasta 300 pg/ml). Probablemente presentará además todo lo que sigue a continuación excepto: 109.

107.

SIMULACRO 3

1. Déficit de 1,25 dihidroxivitamina D. 2. Déficit de 25 hidroxivitamina D. 3. Lesiones óseas típicas de enfermedad de alto remodelado. 4. Aumento del tamaño de la glándula paratiroidea. 5. Aumento del número de receptores de vitamina D y calcio de la glándula paratiroidea.

Se presenta el caso de un adolescente con fiebre en agujas, taquicardia, cefalea, deshidratación, progresivo mal estado general, dolor y aumento de temperatura local, sobre el tercio medio del fémur desde hace aproximadamente 15 días. En la urgencia presenta Cr 5.15 mg/dl, urea 130 mg/dl. Proteinuria en rango no nefrótico. En la radiografía se aprecia patrón apolillado en el fémur. ¿Cómo espera que tenga el complemento el paciente?

Paciente mujer de 64 años de edad sin AP de interés ingresa por deterioro del nivel de conciencia.

1. C3 bajo y C4 normal. 2. C3 normal y C4 normal.

-20-

booksmedicos.org

les, sin resultados hasta el momento. Niega urgencia miccional en ninguna situación.

3. C3 bajo y C4 bajo. 4. C3 alto y C4 normal. 5. C3 alto y C4 alto.

110.

1. Se trata de una incontinencia urinaria de esfuerzo pura, y probablemente la paciente se beneficiará de la colocación de un sling suburetral. 2. Se trata de una incontinencia urinaria de esfuerzo pura, y deberá ser tratada con anticolinérgicos. 3. Se trata de una incontinencia urinaria de esfuerzo, hecho que deberá ser comprobado mediante un estudio urodinámico antes de proponer tratamiento alguno. 4. Se trata de una incontinencia urinaria mixta y deberá ser tratada con anticolinérgicos más sling ureteral. 5. Se trata de una incontinencia urinaria de mixta, hecho que deberá ser comprobado mediante un estudio urodinámico antes de proponer tratamiento alguno.

Un paciente diagnosticado de LES de 5 años de evolución con sintomatología menor, basada en astenia, artralgias y eritema malar con la exposición solar. Acude a la consulta refiriendo mayor astenia y dolor abdominal súbito e intenso. Presenta febrícula 37,5º C. Analíticamente muestra Hb 7 g/dl, Hto 20%, plaquetas 40.000, leucocitos 5200 (fórmula normal), urea 35 mg/dl, Cr 1 mg/ dl. En el estudio de extensión se comprueba la existencia de un ADAMTS 13 suprimido. El paciente:

1. Ha desarrollado un brote de su lupus que afecta a la médula ósea. 2. Ha desarrollado un infarto esplénico que le ocasiona secuestro plaquetario. 3. Ha recibido tratamiento para su brote y como efecto secundario ha desarrollado toxicidad medular. 4. Ha desarrollado una microangiopatía trombótica asociada a su LES. 5. Ha desarrollado una vasculitis asociada a su LES.

111.

113.

Un niño de 8 años acude al servicio de urgencias aquejado de un intenso dolor testicular que se ha iniciado bruscamente al tirarse a la piscina esta mañana. No tiene fiebre. El teste aparece a la exploración horizontalizado y su elevación no disminuye el dolor. No es cierto que:

1. Es un cuadro muy infrecuente en niños. 2. Precisará exploración quirúrgica urgente. 3. Es posible que finalmente precise una orquiectomía si ha pasado mucho tiempo desde la instauración del cuadro. 4. No se resolverá con antibioterapia de amplio espectro aunque demos tratamiento prolongado. 5. Se debe fijar el teste, así como el contralateral.

Un paciente de 70 años sin antecedentes de interés conocidos acude por anuria de 3 días de evolución. Se le realiza un sondaje vesical que descarta un globo vesical puesto que no se obtiene orina. Analíticamente presenta una Cr 6 mg/dl, urea 150 mg/ dl, Na 135 mEq/l, K 4 mEq/l. En la bioquímica de orina tiene Na 135 mEq/l, K 5 mEq/l. ¿Qué esperaría encontrar en la ecografía?

1. Unos riñones aumentados de tamaño. 2. Unos riñones disminuidos de tamaño. 3. Una próstata aumentada de tamaño y vejiga distendida. 4. Una dilatación ureteral bilateral. 5. Una cortical renal adelgazada.

112.

SIMULACRO 3

114.

Una paciente de 67 años acude a la consulta por fugas de orina a lo largo del día que le incomodan muchísimo y condicionan terriblemente su vida ya que se producen con mínimos esfuerzos. Se trata de una paciente muy activa y limita sus clases de baile e incluso hacer la compra. Tiene sobrepeso moderado ya que después de sus 4 partos nunca volvió a su IMC habitual. Varios cultivos de orina han resultado negativos. Ha realizado ejercicios de suelo pélvico e incluso ha utilizado conos vagina-

Un paciente de 76 años acude a su consulta derivado desde el servicio de urgencia, a donde acudió por un cuadro de hematuria autolimitada. Se le realiza una ecografía, donde se aprecia una lesión ocupante de espacio intravesical de unos 2 cm de diámetro, pediculada. Se decide la realización de RTU vesical, tras la que el patólogo informa de un tumor T1 G3, con márgenes libres de tumor, por lo que se decide iniciar terapia con BCG intravesical. Señale la falsa:

1. El tratamiento con BCG previene, o al menos retrasa, el riesgo de progresión tumoral. 2. Los efectos secundarios graves aparecen en menos del 5% de los pacientes. 3. En pacientes con alto riesgo de progresión está indicando un esquema de mantenimiento con BCG. 4. En pacientes en los que ha fallado la BCG se no se debe recomendar la cistectomía.

-21-

booksmedicos.org 5. El esquema óptimo y la duración de la quimioterapia permanece sin aclarar, pero probablemente se debe administrar durante 6-12 meses.

115.

4. La probabilidad de padecer un cáncer de cérvix, si la prueba ha sido positiva, es del 25%. 5. De todos los individuos con cáncer de cérvix, el 25% darán positivo al realizarles la prueba.

Un paciente de 71 años tras la detección de una cifra de PSA de 16 y un tacto rectal sospechoso es diagnosticado de un Ca. de próstata mediante BTRE Gleason 7 (4+3). Tanto la gammagrafía como el TAC abdominopélvico son negativos. El paciente decide la realización de tratamiento con radioterapia, alcanzando un valor de PSA nadir de 1,5 ng/ml. Un año y medio después su PSA es de 1,9 ng/ml. En este caso:

118.

1. Se debe solicitar un estudio de extensión completo. 2. Se debe solicitar únicamente un TAC abdomino pélvico para valorar la posibilidad de una recidiva local. 3. Se debe iniciar bloqueo hormonal completo. 4. Se debe considerar este valor de PSA como normal y continuar el seguimiento como hasta ahora. 5. Se debe plantear dar un nuevo ciclo de RT curativa.

Un paciente de 78 años acude a su consulta por disfunción eréctil. No ha tomado previamente tratamiento alguno. Como antecedentes de interés, refiere toma de antihipertensivos orales, un AIM hace un año y toma de un hipolipemiante que no sabe precisar. Ante la posibilidad de tratar con un PDE5:

120.

2300 500 1800 1300 No se puede calcular con estos datos.

¿Qué tipo de estudio experimental es un ensayo de una intervención preventiva aplicada a una colectividad?

1. 2. 3. 4. 5.

1. Está contraindicada en este caso por el hipolipemiante. 2. Está contraindicada en este caso por el uso del antihipertensivo oral. 3. Está contraindicada por el antecedente personal de infarto agudo de miocardio. 4. Está contraindicado por la edad del paciente. 5. No existe contraindicación aparente.

117.

En una población de 10.000 personas se aplica un test para detección de hipercolesterolemia. Sabemos que la prevalencia de hipercolesterolemia en dicha población es del 10%. Aplicamos un test que considera anormales valores de colesterol en sangre por encima de 220 mg/dl. La sensibilidad de la prueba es del 50% y su especificidad del 80%. ¿Cuántas personas con aumento anormal de colesterol nos dará nuestro test?

1. 2. 3. 4. 5.

119. 116.

SIMULACRO 3

Ensayo clínico cruzado. Estudio de cohortes. Ensayo de campo. Ensayo comunitario de intervención. Ensayo clínico secuencial.

¿Cuál de las siguientes características no corresponde a los estudios de prevalencia?

1. Permiten estudiar varias enfermedades o factores de riesgo a la vez. 2. Se realizan en un corto periodo de tiempo. 3. Ayudan a la planificación sanitaria. 4. La medida de las variables de interés se realizan en un mismo punto del tiempo en todos los individuos. 5. Pueden existir sesgos de supervivencia.

Se aplica una nueva prueba diagnóstica que utiliza una técnica de inmunofluorescencia para la detección precoz del cáncer de cérvix. Cuál de las siguientes opciones es verdadera, sabiendo que el valor predictivo positivo de la misma es del 25%:

1. De cada 100 pruebas realizadas, 25 resultarán verdaderas positivas. 2. La probabilidad de padecer realmente un cáncer de cérvix entre todos los sometidos a la prueba es del 25%. 3. Para encontrar 25 enfermos con cáncer de cérvix es necesario realizar la prueba a 100 individuos.

121.

Las edades de una muestra de 300 pacientes siguen una distribución normal con media 50 años y desviación típica 10 años. Esto implica que:

1. Aproximadamente el 95% de los pacientes tienen edades entre 40 y 60 años.

-22-

booksmedicos.org 2. Existe un 95% de probabilidades de que la verdadera edad media se sitúe entre 40 y 60 años. 3. Existe un 95% de probabilidades de que la verdadera edad media se sitúe entre 30 y 70 años. 4. Aproximadamente el 99% de los pacientes tienen edades entre 40 y 60 años. 5. Aproximadamente el 95% de los pacientes tienen edades entre 30 y 70 años.

122.

125.

Si a usted le comentan que el 76% de los niños de 20 meses tienen un peso de 18 kg o menos, se están refiriendo desde el punto de vista estadístico a:

Una medida de posición llamada rango. Una medida de posición llamada mediana. Una medida de tendencia central llamada media. Una medida de dispersión llamada desviación típica. 5. Una medida de posición llamada percentil.

Un estudio informa que la mediana de superficie cutánea quemada total (SCQT) en una determinada unidad de quemados es del 20%, con una media aritmética del 40%. A la vista de estos datos usted puede interpretar que:

1. La mitad de los pacientes quemados tiene una SCQT mayor del 40%. 2. La mayor parte de los pacientes tienen una SCQT mayor del 20%. 3. La distribución de la SCQT es uniforme. 4. La SCQT más probable es del 40%. 5. La mitad de los pacientes tiene una SCQT inferior al 20%.

124.

126.

Al realizar la estimación poblacional de la media de tensión arterial sistólica pediátrica a partir de una muestra de niños representativa de la población, usted estima que con un 95% de confianza la media de tensión arterial sistólica poblacional se sitúa entre 194 y 206 mmHg. Si usted sabe que la muestra en la que se ha basado para realizar los cálculos es de 122 niños, ¿cuál de entre las siguientes es el valor del parámetro de dispersión de la muestra estudiada?

1. 2. 3. 4. 5.

Una paciente de 18 años consulta con el psiquiatra por que desde hace varios meses le preocupa mucho la posibilidad de contagiarse de una enfermedad con lo que come; revisa las fechas de caducidad de todos los alimentos, llegando incluso a rebuscar en la basura para comprobarlas una y otra vez; también le preocupa poder atragantarse con un trozo de hueso o una espina, por lo que evita comer carne o pescado, desmenuzando los filetes antes de intentar tragarlo. Ha desarrollado una anemia ferropénica por este motivo y se muestra desesperada por su problema. ¿Cuál de las siguientes afirmaciones sobre su trastorno es INCORRECTA?

1. Muchos pacientes acaban acostumbrándose a estas preocupaciones e incorporan los rituales a su vida cotidiana, perdiendo la conciencia de su carácter patológico. 2. Los antidepresivos ISRS son actualmente la medicación de primera elección para esta enfermedad. 3. Con mucha frecuencia estos pacientes desarrollan episodios psicóticos como parte de la evolución natural del trastorno. 4. La terapia cognitivo-conductual es el abordaje psicológico de elección. 5. Esta enfermedad afecta por igual a hombres y mujeres.

1. 2. 3. 4.

123.

SIMULACRO 3

Un varón de 21 años es traído a Urgencias por la Policía Local tras protagonizar un episodio de agitación en un local comercial; se muestra excitado y verborreico, diciendo a gritos que es el nuevo “rey del Rap” y que tiene que viajar a Detroit para grabar con Eminem. La familia informa que lleva una semana más nervioso y sin dormir más que 2-3 horas al día. Dicen que hace un año pasó por unos meses de intenso desánimo coincidiendo con el asesinato de un amigo en una pelea entre bandas locales; no consultó con ningún especialista y regresó a su actividad normal sin recibir tratamiento alguno. ¿Qué podemos decir sobre el trastorno mental que probablemente sufre?

1. Se trata de un trastorno bipolar II, dado que pasó por un episodio depresivo mayor el año pasado y ahora presenta un episodio hipomaníaco. 2. El consumo de drogas nunca va a provocar un cuadro semejante. 3. El antecedente de una “depresión reactiva” permite descartar el diagnóstico de enfermedad bipolar. 4. Si contara alucinaciones auditivas nos inclinaríamos hacia el diagnóstico de esquizofrenia. 5. Sufre actualmente una manía psicótica por lo que será necesario su ingreso involuntario.

11 33 3 6 200

-23-

booksmedicos.org 127.

Consulta un varón, de 65 años de edad, quien desde hace un año tiene unos sueños muy vívidos en los que dice ser perseguido o atacado por desconocidos de los que trata de defenderse o huir; además parece tener un sueño muy inquieto, sobre todo de madrugada, moviéndose de una forma llamativa; la semana pasada se despertó al golpearse contra una silla de su dormitorio; él recuerda que estaba escapando de un atracador y saltó por encima de un seto; en otra ocasión se despertó por el ruido de la lámpara de la mesilla de noche que había lanzado durante el sueño. Su mujer está muy asustada por esto que le sucede y piensa si no estará enloqueciendo. ¿Cuál de las siguientes parasomnias es más probable que presente este paciente?

1. 2. 3. 4. 5.

128.

5. Se trata de un ictus y debería realizarse un TAC craneal urgente.

130.

Un consumidor habitual de cocaína se queja de que nota cómo cientos de insectos le recorren la piel y se meten bajo la misma; esta idea le resulta muy desagradable y parece derivar de la experiencia de un prurito intenso para el que no han encontrado causa aparente. El nombre clásico de este fenómeno es:

1. 2. 3. 4. 5.

Epilepsia hípnica. Sonambulismo. Despertar confusional. Trastorno de la conducta asociado al REM. Terrores nocturnos.

131.

La asistencia ambulatoria de los enfermos psiquiátricos pivota alrededor de los “Centros de Salud Mental”. Son equipos multidisciplinares dedicados en exclusiva a los trastornos mentales. ¿Cuál de los siguientes profesionales NO trabaja en un CSM?

132. A las pocas horas de haberle sido inyectada intramuscularmente una ampolla de haloperidol una paciente de 20 años que parece sufrir un primer episodio psicótico, desarrolla una torsión del cuello hacia la derecha que le resulta dolorosa pero no consigue controlar; además dice notar la lengua “rara”, como “de trapo” y articula deficientemente las palabras. ¿Qué le está sucediendo y cómo se debería tratar?

Síndrome de Ekbom. Síndrome de Briquet. Ftiriasis. Signo de Magnan. Signo de Russell.

Una paciente te comenta en consulta que hace unos días tenía que realizar un viaje en tren a una ciudad y apareció sin saber cómo en otra ciudad, situada en dirección totalmente opuesta; no recuerda nada del viaje pero conserva un billete pagado por ella con su tarjeta de crédito. ¿Cuál de las siguientes categorías define mejor a su problema?

1. 2. 3. 4. 5.

1. Psiquiatra. 2. Neurólogo. 3. Psicólogo clínico. 4. Trabajador social. 5. Enfermero.

129.

SIMULACRO 3

Amnesia psicógena. Abducción idiopática. Fuga disociativa. Ausencia atípica. Personalidad múltiple.

Una organización no gubernamental quiere poner en marcha un programa en un barrio marginal de una ciudad. ¿Qué pasos y qué orden debe seguir para diseñar el programa?

1. Analizar los problemas de salud, priorizar los problemas, movilizar los recursos, definir los objetivos, determinar las actividades, ejecutar el programa y evaluarlo. 2. Definir los objetivos, movilizar los recursos, determinar las actividades, ejecutar el programa y evaluarlo. 3. Analizar los problemas de salud, definir los objetivos, priorizar los objetivos, determinar las actividades, movilizar los recursos, ejecutar el programa y evaluarlo. 4. Analizar los problemas de salud, priorizar los problemas, determinar los objetivos, determinar las actividades, movilizar los recursos, ejecutar el programa y evaluarlo.

1. Es un fenómeno catatónico y debería recurrirse a la EC. 2. La administración de biperideno resolverá el síntoma rápidamente. 3. Los pacientes ancianos son especialmente proclives a sufrir este problema. 4. La aparición de estos movimientos anormales sugiere un origen neurológico de la psicosis.

-24-

booksmedicos.org 5. Definir los objetivos, priorizar los objetivos, movilizar los recursos, determinar las actividades, ejecutar el programa y evaluarlo.

133.

134.

Análisis coste-beneficio. Análisis coste-efectividad. Análisis coste-utilidad. Minimización de costes. Maximización de la efectividad.

137.

0,26 7,50 0,36 5,86 8,10

Un paciente de 52 años de edad con cáncer de pulmón metastásico en situación terminal recibe tratamiento habitual con morfina oral de liberación retardada 30 mg cada 12 horas. En este momento se encuentra en situación de agonía y no puede deglutir. ¿Cuál sería la dosis adecuada de morfina subcutánea en esta situación, teniendo en cuenta que su función renal previa era normal?

1. 2. 3. 4. 5.

¿Cuál será el número de camas de hospitalización previsto en un año (365 días) para una zona de 100.000 habitantes que presenta una frecuentación hospitalaria de 100 por año y 1.000 habitantes, con una estancia media de 10 días y un índice o porcentaje de ocupación del 90%?

1. 2. 3. 4. 5.

136.

1. Balón de contrapulsación-aórtico. 2. Cateterismo cardíaco con agiografía coronaria y ventriculografía izquierda. 3. Catéter de Swan-Ganz para valorar mejor el tratamiento diurético. 4. Morfina intravenosa. 5. Envío a su domicilio, en su pueblo, para que fallezca allí.

Si un servicio de endocrinología ha presentado un número de 20 primeras consultas y 150 visitas sucesivas durante el mes de febrero de 1996, ¿cuál es el índice de consultas?

1. 2. 3. 4. 5.

135.

disnea muy intensa secundaria a un edema agudo de pulmón. ¿Cuál sería la conducta más adecuada a seguir?

¿Qué tipo de evaluación económica es el proceso de identificar y cuantificar los costes de procedimientos alternativos para alcanzar un objetivo cuyas consecuencias se suponen similares?

1. 2. 3. 4. 5.

SIMULACRO 3

138.

300 305 310 315 500

Varón de 57 años diagnosticado de cáncer epidermoide de laringe en progresión. Ha recibido tratamiento quirúrgico y radioterápico. Actualmente refiere molestias leves e inespecíficas en región mandibular izquierda con crisis paroxísticas (latigazos) de dolor muy intenso en la misma región, de segundos de duración. Estas crisis aparecen aproximadamente 10 veces al día y NO se relacionan con nada. El tratamiento farmacológico habría que iniciarlo con:

1. 2. 3. 4. 5.

Anciano de 95 años de edad con pluripatología senil, diagnosticado de insuficiencia cardíaca congestiva en fase terminal por lo que ha precisado de múltiples ingresos hospitalarios, habiéndose demostrado en uno de ellos, mediante ecocardiografía, una fracción de eyección ventricular inferior al 20%. Tras una semana de ingreso hospitalario con tratamiento adecuado con oxígeno, vasodilatadores, diuréticos y fármacos inotrópicos positivos parenterales, el paciente se encuentra en situación de fracaso multiorgánico, y presenta una

139.

-25-

30 mg cada 12 horas. 5 mg cada 4 horas. 10 mg cada 8 horas. 15 mg cada 6 horas. 30 mg cada 4 horas.

Opiáceos potentes. AINE combinados con opiáceos débiles. AINE solos. Antidepresivos y/o anticonvulsivantes. Corticoides.

Un paciente de 53 años es intervenido de hemorroides grado IV realizándose una hemorroidectomía de 2 grupos hemorroidales. De las siguiente, indique cuál considera mejor alternativa terapéutica para tratar el dolor postoperatorio:

booksmedicos.org

que aparecieron después del dolor. La exploración es difícil por la poca colaboración de la paciente, que está muy nerviosa y por la obesidad que presenta, pero parece focalizar el dolor en FID con Blomberg positivo. En la analítica destaca leucocitosis con desviación izquierda y elevación de PCR. Niega relaciones sexuales, no obstante es sometida a una ecografía ginecológica donde sólo se aprecia pequeña cantidad de líquido libre en Douglas. ¿Cuál le parece la mejor opción en este caso?

1. Se trata de un dolor postoperatorio leve por lo que sería suficiente con la administración de paracetamol cada 6 horas a demanda. 2. Se trata de un dolor moderado por lo que convendría iniciar la analgesia con antiinflamatorios. 3. Se trata de un dolor severo por lo que pondría de entrada morfina. 4. Se trata de un dolor severo por lo que convendría asociar paracetamosl con AINE. 5. La infiltración con anestésicos locales de la herida sería suficiente para controlar el dolor.

140.

1. Alta y revisión de nuevo en 24 horas. 2. Mantener en observación hospitalaria para ver evolución. 3. Laparotomía porque parece que tiene una apendicitis aguda. 4. Laparoscopia diagnóstica y actuar según los hallazgos. 5. Ingreso con antibioterapia intravenosa.

A un paciente de 34 años se le indica cirugía de reflujo gastroesofágico por presentar esofagitis grado II. Con respecto a la funduplicatura tipo Nissen que se le va a realizar, una de las siguientes afirmaciones no es correcta:

1. 2. 3. 4.

El abordaje de elección es la vía laparoscópica. El fundus gástrico rodea al esófago 180º. Si queda muy apretada, puede producir disfagia. Si queda muy laxa, la sintomatología puede recidivar. 5. Si el paciente presentara un esófago corto, asociaríamos una gastroplastia de Collis.

141.

142.

143.

Un paciente varón de 41 años presenta una colitis ulcerosa corticodependiente refractaria al tratamiento médico por lo que se indica cirugía. Es intervenido de manera programada realizándose una panproctocolectomia con creación de reservorio ileoanal e ileostomía de protección. El postoperatorio trascurre con normalidad siendo dado de alta sin incidencias. Dada la buena evolución se decide cerrar la ileostomía a los 2 meses de la primera intervención. Dicha intervención cursa también sin complicaciones y el paciente manifiesta una buena continencia con un ritmo defecatorio que él considera aceptable. Dos meses más tarde comienza con rectorragia, aumento de su frecuencia defecatoria, dolor abdominal y febrícula. ¿Qué entidad hay que descartar como primera opción en este paciente?

1. 2. 3. 4. 5.

SIMULACRO 3

Mujer de 26 años que acude a consulta por disfagia intermitente a sólidos desde hace 1 año. No refería pérdida de peso aunque si dolor retroesternal ocasional leve. La endoscopia oral fue normal, con mínima irregularidad en esófago medio del que se toma una biopsia y en el que se observa una mucosa normal con 55 eosinófilos por campo de gran aumento en las mismas. La analítica era la siguiente: Hb 12.6 g/dl, 6.300 leucocitos/mm3 con neutrófilos 65% y linfocitos 28%, plaquetas 280.000/mm3, creatinina 0.6, urea 23, GOT 19, GPT 28, GGT 33, bilirrubina 0.6, sodio 142, potasio 4.1. Acerca del diagnóstico que usted sospecha, ¿cuál es el tratamiento más adecuado en este momento?

1. 2. 3. 4. 5.

144.

Gastroenteritis aguda. Recidiva de su colitis sobre el intestino delgado. Una reservoritis. Absceso intraabdominal. Absceso isquiorrectal.

Fluticasona oral. Nistatina oral. Rifaximina oral. Almagato oral. Ciclosporina i.v.

Paciente de 23 años que refiere malestar postpandrial con dolor leve de tipo cólico, digestión pesada y sensación de distensión abdominal. Acerca del cuadro que presenta el paciente, señale de entre las siguientes afirmaciones, cuál es cierta:

1. Se realizará gastroscopia para descartar organicidad. 2. La asociación a H. pylori es excepcional, por lo que no es necesario investigar ni erradicar en caso de ser positivo. 3. Si presenta estreñimiento asociado, el diagnóstico más probable hace que deba descartarse una obstrucción intestinal con colonoscopia.

Una mujer de 24 años acude a urgencias por dolor abdominal de 24 horas de evolución localizado en fosa iliaca derecha (FID), acompañado de vómitos

-26-

booksmedicos.org

lítica fue normal. Se realiza una ecografía abdominal observándose en transcavidad de los epiplones y comprimiendo tercera porción duodenal una colección de 7 cm. ¿Cuál sería la actitud más adecuada?

4. El tratamiento con magaldrato mejora prácticamente a la totalidad de los pacientes. 5. No tiene datos de alarma, lo que permite el tratamiento directo de los pacientes sin realizar más pruebas complementarias.

145.

146.

148.

Trasplante hepático. Tratamiento con sorafenib. Quimiembolización. Ablación por radiofrecuencia. Cirugía resectiva.

149.

Neoplasia quística pancreática. Pseudoquiste pancreático. Quiste seroso pancreático. Ascitis pancreática. Adenocarcinoma de colon perforado.

Mujer de 54 años que acude a consulta al mes del alta, tras ingreso hospitalario por una pancreatitis aguda litiásica por vómitos de repetición. La ana-

Tratamiento antibiótico. Drenaje transgástrico por ecoendoscopia. Suplemento con enzimas pancreáticos. Metoclopropamida oral. Duodenopancreatectomía cefálica de Whipple.

Varón de 56 años que acude a urgencias por dolor en epigastrio que irradia hacia hipocondrio derecho, nauseas y vómitos alimentario-biliosos e ictericia leve. La analítica realizada en urgencias fue la siguiente: Hb 14.2 g/dl, leucocitos 11.000/ mm3 con 80% neutrófilos, plaquetas 200.000/mm3, glucosa 123 mg/dl, urea 45 mg/dl, creatinina 1.0 mg/dl, albúmina 5.5 g/dl, GOT 87 U/l, GPT 89 U/l, GGT 88 U/L, bilirrubina total 2.5 mg/dl, fosfatasa alcalina 400 U/l, LDH 120 U/l, amilasa 15 U/ ml, sodio 136, potasio 4.8. Se realiza una ecografía abdominal donde se observa una vesícula con una imagen hiperecogénica en el infundíbulo en íntimo contacto con colédoco, colédoco distal de 5 mm con dilatación proximal desde hepático común y de vía biliar intrahepática. ¿Cuál de entre las siguientes, es el diagnóstico más probable?

1. 2. 3. 4. 5.

Varón de 55 años, bebedor de 70 g/día de alcohol y fumador, acude a su consulta por dolor epigástrico leve y náuseas. La exploración física demuestra dolor abdominal leve en piso abdominal superior y telangiectasias en tórax. La analítica fue normal. Se realiza una ecografía abdominal observándose en la cabeza de páncreas una colección de 9 cm con bordes irregulares. Una punción por ecoendoscopia demostró liquido filante y los siguientes valores: leucocitos 100/mm3, amilasa 30 U/l, proteínas 3 mg/dl, mucina +, CEA 290 ng/ml. ¿Cuál es el diagnóstico más probable?

1. 2. 3. 4. 5.

147.

1. 2. 3. 4. 5.

Varón de 50 años que acude a su consulta por astenia intensa e ictericia leve. La exploración física únicamente reveló hepatomegalia leve. La analítica fue: Hb 15.2 g/dl, VCM 82.5 fl, leucocitos 9.000/ mm3 con 55% neutrófilos, plaquetas 168.000/mm3, glucosa 101 mg/dl, urea 50 mg/dl, creatinina 0.9 mg/dl, albúmina 3.9 g/dl, GOT 200 U/l, GPT 100 U/l, GGT 198 U/l, bilirrubina total 3.8 mg/dl, fosfatasa alcalina 498 U/l, LDH 587 U/l, sodio 139, potasio 4.8, antiHBs -, AgHBs +, AntiHBc +, AgHBe -, antiHBe +, DNA -VHB +, anti-VHC -, AgVHD -.Se realiza una ecografía abdominal en la que se observa un hígado heterogéneo e irregular con una lesión nodular de 4.5 cm que invade porta y colédoco. Se realiza una RMN donde se observan los mismos hallazgos, siendo una lesión con realce en fase arterial y lavado precoz en fase venosa. ¿Cuál sería la actitud más adecuada en este paciente?

1. 2. 3. 4. 5.

SIMULACRO 3

Ampuloma. Coledocolitiasis. Colangiocarcinoma intrahepático. Síndrome de Mirizzi. Enfermedad de Caroli.

Varón de 49 años, bebedor de 50 g/día de alcohol y fumador de 20 cig/día que acude a su consulta por dolor abdominal y diarrea de 3 meses de evolución. La exploración física demostró la existencia de dolor abdominal en hemiabdomen superior a la palpación sin peritonismo. La radiografía simple de abdomen mostraba un patrón gaseoso inespecífico con pequeñas calcificaciones en región central de abdomen superior. La analítica realizada fue la siguiente: Hb 12.2 g/dl, leucocitos 7.900/mm3, plaquetas 159.000/mm3, glucosa 103 mg/dl, urea 55 mg/ dl, creatinina 0.9 mg/dl, albúmina 5.1 g/dl, GOT 97 U/l, GPT 40 U/l, GGT 65 U/l, bilirrubina total 0.7 mg/dl, fosfatasa alcalina 180 U/l, LDH 100 U/l, amilasa 8 U/ml, sodio 140, potasio 4.¿Cuál sería el tratamiento más adecuado para tratar la diarrea?

1. Infliximab. 2. Pancreatina.

-27-

booksmedicos.org 153.

3. Loperamida. 4. Mesalazina. 5. Metronidazol.

150.

Mujer de 40 años que presenta diarrea de hasta 10 deposiciones al día de dos meses de evolución. La grasa en heces de 24 horas es de 9 gramos. Se realizó un test de D-xilosa con niveles bajos en orina a las 5 horas. La alfa-1-antitripsina fecal es alta. ¿Cuál de entre los siguientes es el diagnóstico más probable?

1. 2. 3. 4. 5.

151.

154.

En relación a la absorción de nutrientes a nivel intestinal, señale cuál de las siguientes afirmaciones es INCORRECTA:

1. Los hidratos de carbono se absorben principalmente en intestino delgado proximal. 2. Los triglicéridos de cadena media no precisan de sales biliares para su absorción. 3. La vitamina B12 se une al factor intrínseco en el estómago, siendo imprescindible para su adecuada absorción. 4. Las proteínas se absorben principalmente en el intestino medio. 5. El hierro y el calcio se absorben principalmente en intestino delgado proximal.

Varón de 40 años que acude a su consulta por sensación de plenitud y digestiones pesadas sin otra sintomatología acompañante desde hace 1 mes. La analítica es la siguiente: Hb 11.2 g/dl, VCM 60 fl, HCM 21, plaquetas 280.000/mm3, leucocitos 9.000, glucosa 88 mg/dl, urea 30 mg/dl, creatinina 1,0 mg/dl, bilirrubina total 0,4 mg/dl, GOT 13 U/l, GPT 22 U/l, GGT 21 U/l, fosfatasa alcalina 54 U/l, amilasa 28 U/l, LDH 59 U/l, PCR 3.9 mg/l. ¿Cuál sería la actitud más adecuada a continuación?

1. Tratamiento empírico con omeprazol. 2. Investigar infección por H. pylori y erradicar si es positivo. 3. Gastroscopia. 4. Tratamiento con pantoprazol + domperidona. 5. Tratamiento con cinitaprida.

Sueroterapia. Albúmina i.v. Restricción hídrica. Paracentesis diagnóstica. Enemas lactulosa.

Varón de 34 años alcohólico, que acude a urgencias por ictericia y deterioro del estado general. La exploración física muestra hepatomegalia dura y dolor abdominal leve de forma difusa. La analítica en urgencias es la siguiente: Hb 14.2 g/dl, plaquetas 180.000/mm3, leucocitos 12.000 con 9000 neutrófilos, glucosa 92 mg/dl, urea 30 mg/dl, creatinina 1,0 mg/dl, bilirrubina total 7.4 mg/dl, INR 2.5, GOT 263 U/l, GPT 108 U/l, GGT 381 U/l, fosfatasa alcalina 150 U/l, amilasa 11 U/l, LDH 149 U/l, PCR 8.0 mg/l, sodio 140, potasio 4.1. ¿Cuál de entre las siguientes NO sería una opción adecuada para el diagnóstico más probable?

1. 2. 3. 4. 5.

-28-

Doblar la dosis de IBP. Manometría esofágica. Sucralfato oral. Añadiría domperidona. Prednisona oral

Varón de 58 años diagnosticado de cirrosis por VHC que acude a urgencias por deterioro del nivel de conciencia de 5 horas de evolución. La exploración física mostraba hepatomegalia leve y matidez cambiante en flancos. La analítica en urgencias fue la siguiente: Hb 13.2 g/dl, plaquetas 80.000/mm3, leucocitos 9.000, glucosa 88 mg/ dl, urea 60 mg/dl, creatinina 1,5 mg/dl, bilirrubina total 2.4 mg/dl, GOT 63 U/l, GPT 72 U/l, GGT 81 U/l, fosfatasa alcalina 154 U/l, amilasa 29 U/l, LDH 119 U/l, PCR 3.0 mg/l, sodio 128, potasio 4.3. ¿Cuál de entre las siguientes NO sería una opción aconsejada?

1. 2. 3. 4. 5.

155. 152.

Varón de 64 años, exfumador, EPOC y diabético insulinodependiente, acude a usted por pirosis y molestias retroesternales. No ha mejorado con omeprazol 20 mg/día vía oral. En una endoscopia oral no se observan hallazgos relevantes, salvo abundante reflujo de líquido bilioso. La pHmetría fue normal. ¿Cuál sería el siguiente paso que seguiría en esta situación?

1. 2. 3. 4. 5.

Enfermedad de Whipple. Sobrecrecimiento bacteriano. Insuficiencia pancreática. Esprue colágeno. Esclerodermia.

SIMULACRO 3

Dieta baja en proteínas. Pentoxifilina oral. Prednisona oral. Fitomenadiona i.v. Complejo vitamínico B.

booksmedicos.org 156.

1. 2. 3. 4. 5.

157.

160.

Señale, de entre las siguientes afirmaciones acerca de la hepatitis autoinmune tipo 1, cuál es INCORRECTA:

Es una entidad más frecuente en mujeres. Suele tener anticuerpos antimúsculo liso +. El cociente GOT/FA es < 1. Puede aparecer a cualquier edad. El dato histológico típico es una hepatitis de la interfase.

161.

158.

Penicilina G. Alopurinol. Estreptomicina. Resincolestiramina. N-acetilcisteína.

Niña de 8 meses de edad que presenta retraso del crecimimento, linfadenitis supurativa en cadenas linfáticas inguinales, y abscesos perianales. En el cultivo de los exudados procedentes de las adenopatías se cultiva S. aureus. ¿Cuál es la opción falsa?

1. En el diagnóstico de esta paciente nos sería útil la realización de pruebas de función fagocítica. 2. Está descartada la posibilidad de una enfermedad granulomatosa crónica al tratarse de un individuo del sexo femenino. 3. La prueba del NBT (nitro-azul de tetrazolio), aunque actualmente en desuso era fundamental para descartar esta enfermedad. 4. El número de linfocitos suele no estar alterado. 5. Es frecuente que exista esplenomegalia del diagnóstico.

En un paciente alcohólico, el diagnóstico anatomopatológico es hepatitis alcohólica. ¿Qué hallazgo histológico NO debería encontrar?

1. Necrosis hepatocitaria con reacción neutrofílica. 2. Hialina de Mallory. 3. Esteatosis y nódulos de regeneración. 4. Fibrosis alrededor de la vena centrolobulillar. 5. Acúmulo de hierro en las células de Kupffer.

162. 159.

Una mujer de 40 años se queja de picor generalizado. Un estudio analítico demuestra una elevación del colesterol y la fosfatasa alcalina. Los anticuerpos antimicrosomales están aumentados. Si se realizara una biopsia hepática en este momento inicial de la enfermedad, qué lesión prácticamente diagnóstica se podría observar:

1. Cirrosis. 2. Reacción granulomatosa epitelioide que destruye el conducto biliar. 3. Colestasis. 4. Cuerpos de Mallory. 5. Necrosis de hepatocitos periportales.

Señale cuál de entre los siguientes es el tratamiento más adecuado para un paciente con ingesta de 14 gramos de paracetamol.

1. 2. 3. 4. 5.

SIMULACRO 3

Varón de 19 años de edad que consulta por adenopatías cervicales de 2.5 cm de diámetro. Realiza una biopsia de una de ellas en la que se objetiva un patrón nodular con presencia de áreas difusas, y borramiento de la arquitectura ganglionar, con abundantes linfocitos B, acompañados de células gigantes con núcleos multilobulados y arrugados, con aspecto en “palomitas de maíz” y nucléolos basófilos, rodeados de una corona de linfocitos T. La enfermedad que padece este paciente es:

Niño de 2 meses, que presenta muguet y neumonía por CMV. En la exploración destaca la falta de medro y ausencia aparente de amígdalas. Ante estos datos ¿qué sospecharía usted?

1. Puede formar parte de la evolución normal del lactante. 2. Sd. de Bruton aunque harían falta más pruebas como la cuantificación de inmunglobulinas y de subpoblaciones linfocitarias. 3. Sd. hiper-IgM. 4. Enfermedad granulomatosa crónica. 5. Inmunodeficiencia combinada grave o severa.

1. Mononucleosis infecciosa. 2. Linfadenopatía generalizada persistente asociada a infección por VIH. 3. Linfoma del manto. 4. Enfermedad de Hodgkin. 5. Linfoma Burkitt.

163.

-29-

Aunque la principal función del complemento es provocar la lisis bacteriana, su activación tiene implicación en otros procesos inmunológicos, como la inflamación y la quimiotaxis. ¿Cuál de las si-

booksmedicos.org guientes moléculas derivadas de la activación del complemento tiene una importante función quimiotáctica?

1. 2. 3. 4. 5.

164.

167.

C5a. C4d. C3b. C2a. FH.

168. 1. 2. 3. 4. 5.

165.

166.

Fenocopia. Genocopia. Heterogenicidad genética de alelo. Heterogenicidad genética de locus. Heterogenicidad clínica.

Un paciente presenta normalidad de los gases sanguíneos en reposo e hipoxemia con aumento del gradiente alveoloarterial de oxígeno, sin hipercapnia, cuando realiza ejercicio físico. Si respira una mezcla rica en oxígeno, la hipoxemia se corrige. ¿Qué tipo de alteración de la función respiratoria presenta el paciente?

Trastorno ventilatorio obstructivo. Trastorno ventilatorio restrictivo. Alteración pura de la difusión. Alteración de relación ventilación/perfusión por efecto shunt. 5. Alteración de la relación ventilación/perfusión por efecto aumento del espacio muerto.

Los aminoglicósidos. Las antracilinas (daunomicina, adriamicina). Los antibióticos polipeptídicos. Las quinolonas. Los fungostáticos imidazólicos.

169.

Tras la administración de una inyección intramuscular de bencil-penicilina a un enfermo del que no se conocían sus antecedentes alérgicos a esta sustancia, aparecen bruscamente y de forma inmediata tras la administración: enrojecimiento generalizado, síntomas de asfixia por broncoconstricción y angustia. ¿Cuál de los siguientes fármacos elegiría para su inmediata administración?

Varón de 35 años residente en Valencia, sin enfermedad cardiopulmonar previamente diagnosticada, que viaja a Perú, pasando, por tanto, bruscamente de estar al nivel del mar a estar a 2400 m de altitud. ¿Qué cambio fisiológico presentará en minutos-horas?

1. 2. 3. 4. 5.

170. 1. 2. 3. 4. 5.

Paration. Metacolina. Atropina por vía i.v. Fisostigmina. Dopamina.

1. 2. 3. 4.

Indique cuál de los siguientes grupos de antimicrobianos tiene una acción inhibidora de la DNAgirasa (que explica sus acciones antibacterianas, sobre la incorporación de timidina en linfocitos humanos y sobre algunas topoisomerasas II de células eucariotas):

1. 2. 3. 4. 5.

Ante un enfermo que ingresa en un Servicio de Urgencia con náuseas, vómitos, dolor epigástrico, salivación, dolor subesternal, disnea, sudoración y que la exploración manifiesta bloqueo de conducción cardíaca, sudoración, dificultad de acomodación y en quien, como antecedente, figura una excursión campestre y la ingestión de setas 1 hora antes de aparecer la sintomatología. Aparte de las medidas generales, de mantenimiento, instauraría un tratamiento con:

1. 2. 3. 4. 5.

La fibrosis quística es una enfermedad de herencia autosómica recesiva, causada por mutaciones en el gen CFTR. Se han descrito más de 1500 mutaciones en dicho gen, causantes de la enfermedad. Este es un claro ejemplo de:

SIMULACRO 3

Dopamina. Dexametasona. Prednisona. Prednisolona. Adrenalina.

-30-

Policitemia. Aumento de la PCO2 alveolar. Bradicardia. Disminución del flujo sanguíneo cerebral. Hiperventilación.

Mujer de 20 años, consulta por malestar general, debilidad muscular y calambres. En la analítica se evidencia función renal: normal; alcalosis metabólica e hipopotasemia. Los iones en orina son los siguientes: Na: 20 mEq/l; K: 55 mEq/l; Cl: 0 mEq/l; HCO3: 70 mEq/l. ¿Cuál es la etiología más probable de la alcalosis metabólica?

booksmedicos.org 1. 2. 3. 4. 5.

171.

En un paciente con hiponatremia, osmolaridad sérica disminuida, concentración de sodio en orina mayor de 20 mmol/l y datos clínicos de normalidad del volumen del espacio extracelular, ¿cuál de las siguientes es la causa más probable?

1. 2. 3. 4. 5.

172.

1. 2. 3. 4. 5.

Vómitos subrepticios. Síndrome de Bartter. Hiperaldosteronismo primario. Estenosis de la arteria renal. Síndrome de Cushing.

174.

Déficit de glucocorticoides. Hiperglucemia severa. Déficit de mineralocorticoides. Uso encubierto de diuréticos. Insuficiencia renal aguda.

175.

1. La potencia de contracción del músculo esquelético depende principalmente de la acción de los iones calcio liberados desde el retículo endoplásmico de la propia célula. 2. La potencia de contracción del músculo esquelético depende principalmente de la acción de los iones calcio de los líquidos extracelulares. 3. En el músculo cardíaco, pero no en el esquelético, disminuye la permeabilidad al potasio inmediatamente después de iniciarse el potencial de acción. 4. En el músculo cardíaco el potencial de membrana de reposo se recupera gracias a la salida de Na+ al espacio extracelular. 5. El potencial de acción del músculo esquelético depende totalmente de la apertura de canales rápidos de Na+, interviniendo además en el cardíaco, canales lentos de Na+ y Ca2+.

173.

177.

-31-

Hipotiroidismo. Esferocitosis hereditaria. Síndrome mielodisplásico. Anemia inmunohemolítica. Anemia de enfermedad crónica.

Mujer de 58 años que consulta por debilidad y astenia crónicas. Su hemograma es el siguiente: leucocitos 2.3 x 109/l, hemoglobina 6 g/dl, plaquetas 80 x 109/l, reticulocitos 30 x 109/l. La LDH sérica es de 1800 U/l. ¿Cuál es el diagnóstico más probable entre los siguientes?

1. 2. 3. 4. 5.

Mujer de 24 años en estado de gestación que refiere aparición de debilidad intensa reciente. La exploración física no revela anomalías excepto llamativa palidez de piel y mucosas y taquicardia. El hemograma presenta leucocitos 1.2 x 109/l, hemoglobina 5 g/dl y plaquetas 30 x 109/l. El aspirado de médula ósea es muy hipocelular. ¿Cuál es el diagnóstico más probable?

Enfermedad celíaca. Hemoglobinuria paroxística nocturna. Anemia sideroblástica. Polimenorrea. Aclorhidria.

Mujer de 30 años que consulta por debilidad de años de duración, nunca estudiada, y aporta el siguiente hemograma: leucocitos 6.5 x 109/l con fórmula normal, hemoglobina 7 g/dl, VCM 73 fl, plaquetas 210 x 109/l, reticulocitos 180 x 109/l. ¿Cuál es el diagnóstico más probable entre los siguientes?

1. 2. 3. 4. 5.

176.

Leucemia aguda mieloblástica. Leucemia aguda linfoblástica. Anemia megaloblástica por deficencia de folato. Hemoglobinuria paroxística clásica. Aplasia de médula ósea.

Enferma de 35 años que refiere debilidad progresiva de varios meses de evolución. En el hemograma hay los siguientes datos: leucocitos 6.5 x 109/l con fórmula normal, hemoglobina 7 g/dl, VCM 74 fl, plaquetas 150 x 109/l. La cifra de ferritina sérica es de 10 microg/l. ¿Cuál es el diagnóstico menos probable?

1. 2. 3. 4. 5.

Las propiedades de las membranas del músculo cardíaco y el músculo esquelético tienen ciertas características diferentes. De las siguientes afirmaciones respecto a este punto, señale la que le parece INCORRECTA:

SIMULACRO 3

Síndrome mielodisplásico. Anemia megaloblástica. Anemia inmunohemolítica. Aplasia de médula ósea. Anemia sideroblástica.

Paciente de 34 años ingresado por neumonía comunitaria y tratado con antibióticos. Durante su ingreso desarrolla anemia aguda sin evidencia de

booksmedicos.org

3. Hidroxiurea. 4. Hidroxiurea más salicilatos. 5. Trasplante de progenitores hematopoyéticos.

sangrado y presenta orinas oscuras. ¿Cuál es la explicación menos probable?

1. 2. 3. 4. 5.

178.

Anemia inmunohemolítica por fármacos. Deficiencia de glucosa-6-fosfato deshidrogenasa. Coagulación intravascular diseminada. Anemia inmunohemolítica por crioaglutininas. Esferocitosis hereditaria.

181.

Una mujer de 38 años, con historia de astenia crónica y episodios de orinas oscuras, es ingresada por cuadro de trombosis venosa profunda para estudio y tratamiento. El hemograma presenta pancitopenia. Entre las pruebas diagnósticas se realiza una citometría de flujo de las células sanguíneas que demuestra células deficientes en antígeno CD59. En relación con el tratamiento de la enfermedad de la paciente, serán útiles todos los siguientes fármacos con la excepción de:

179.

180.

Hierro. Esteroides. Eculizumab. Heparina. Vitamina B12.

Enfermo de 78 años que presenta debilidad progresiva. Su hemograma es el siguiente: leucocitos 5.5 x 109/l, hemoglobina 7 g/dl, VCM 105 fl, plaquetas 50 x 109/l, reticulocitos 30 x 109/l. El estudio de médula ósea demuestra un 8% de blastos. ¿Cuál es el diagnóstico del paciente?

1. 2. 3. 4. 5.

Paciente de 50 años sin antecedentes patológicos de interés. Se remite a consulta para estudio de hemograma patológico con cifra de hemoglobina de 19 g/dl. La saturación arterial de O2 es de 94% La exploración física es normal. Entre los posibles diagnósticos se encuentran todos los siguientes excepto:

1. 2. 3. 4. 5.

182. 1. 2. 3. 4. 5.

SIMULACRO 3

Varón de 60 años que consulta por debilidad progresiva reciente, frecuentes gingivorragias y ocasionales epístaxis. En la exploración se observan encías muy inflamadas y petequias distales en miembros inferiores, sin organomegalias. Su hemograma es el siguiente: hemoglobina 7 /dl, leucocitos 3.5 x 109/l, 60% neutrófilos, 2% eosinófilos, 13% linfocitos, 25% blastos, plaquetas 18 x 109/l. ¿Cuál es el diagnóstico más probable del paciente?

1. 2. 3. 4. 5.

Anemia refractaria sideroblástica. Anemia refractaria con exceso de blastos 1. Anemia refractaria con exceso de blastos 2. Leucemia aguda mieloblástica. Anemia megaloblástica.

183.

Varón de 52 años sin factores de riesgo vascular, que consulta por episodios de mareos y cefalea en los meses previos. Ha perdido peso en cuantía moderada en este tiempo. En la exploración física se detecta esplenomegalia y el hemograma es el siguiente: leucocitos 16 x 109/l, hemoglobina 20 g/ dl, VCM 78 fl, plaquetas 450 x 109/l. En referencia al diagnóstico más probable del paciente, el tratamiento más adecuado será:

184.

-32-

Leucemia aguda linfoblástica T. Leucemia aguda linfoblástica B. Leucemia aguda mieloide promielocítica. Leucemia aguda mieloide monoblástica. Síndrome mielodisplásico con exceso de blastos.

Enferma de 20 años, remitida para estudio de adenopatías cervicales derechas de varias semanas de evolución. La paciente se encuentra asintomática y el resto de la exploración física es normal. ¿Cuál es el diagnóstico más probable entre los siguientes?

1. 2. 3. 4. 5.

1. Flebotomías periódicas. 2. Flebotomías más antiagregantes plaquetarios.

Hemoglobina de baja afinidad por el O2. Policitemia de Chuvash. Poliquistosis renal. Hemangioblastoma cerebeloso. Mioma uterino.

Leucemia linfática crónica. Enfermedad de Hodgkin. Leucemia aguda linfoblástica. Tricoleucemia. Linfoma no hodgkiniano folicular.

Enferma de 25 años que consulta por pérdida de peso reciente y debilidad. La exploración física

booksmedicos.org es normal, excepto ligera palidez. El hemograma presenta una hemoglobina de 10 g/dl y una VSG de 80 mm/h, siendo el resto de resultados normales. En la radiografía torácica se aprecia ensanchamiento mediastínico. ¿Cuál es el diagnóstico más probable?

1. 2. 3. 4. 5.

185.

188.

Paciente de 35 años que consulta por fiebre prolongada y deterioro del estado general. En la exploración física se palpa esplenomegalia. Tras diversos estudios sin resultados diagnósticos, se practica una biopsia de médula ósea donde se visualizan células grandes, binucleadas, CD15 y CD30 positivas. El diagnóstico es:

189.

En un paciente de 38 años diagnosticado de asma bronquial y que presenta insuficiencia respiratoria nasal bilateral persistente con rinorrea e hiposmia debe considerar el diagnóstico de:

1. 2. 3. 4. 5.

Varón de 17 años que acude al médico por presentar ictericia, acompañada de astenia y anorexia. En la analítica presenta una ASAT de 870 UI y una ALAT de 1320 UI, así como una bilirrubina sérica de 6 mg/dl. Estudio serológico: HBsAg+, IgM antiHBc+ y resto de las pruebas normales. ¿Cuál de los siguientes hallazgos morfológicos NO esperaría encontrar si se hiciera una biopsia hepática a este paciente?

1. Cuerpos de Councilman (degeneración acidófila de los hepatocitos). 2. Formación de rosetas o pseudoacinos. 3. Necrosis hepática en puentes. 4. Hipertrofia e hiperplasia de las células de Kupffer. 5. Hepatocitos con citoplasma esmerilado o en vidrio deslustrado.

Ocena. Mucormicosis rinocerebral. Desviación septal. Rinosinusitis crónica con pólipos. Mucocele frontal. 190.

187.

Un niño de 10 años está siendo estudiado por retraso ponderoestatural y diarrea crónica con esteatorrea. Presenta anticuerpos antigliadina, por lo que se decide hacer una biopsia de yeyuno. ¿Cuál de estos patrones esperaría encontrar en la biopsia?

1. Infiltrado linfocitario de la lámina propia con linfáticos y vasos quilíferos dilatados; vellosidades en forma de maza. 2. Vellosidades aplanadas o ausentes y aumento de la infiltración linfocitaria, de predominio distal por la afectación de las placas de Peyer. 3. Infiltrado inflamatorio en la lámina propia y el epitelio intestinal, con hiperplasia criptal y lesión del epitelio de superficie. 4. Acortamiento de las vellosidades con disminución del número de mitosis en las criptas e infiltrado mononuclear en la lámina propia. 5. Lesión parcheada de las vellosidades con aumento de la infiltración linfocitaria en la lámina propia, pero respetando el epitelio.

Linfoma esclerosante mediastínico. Leucemia linfoblástica T. Linfoma no hodgkiniano de linfoblastos T. Enfermedad de Hodgkin. Linfoma no hodgkiniano folicular.

1. Enfermedad de Hodgkin estadio IVBS. 2. Enfermedad de Hodgkin estadio IIIBS. 3. Linfoma no hodgkiniano de células grandes estadio IVBS. 4. Linfoma no hodgkiniano de células grandes estadio IIIBS. 5. Enfermedad de Hodgkin estadio IIBS.

186.

SIMULACRO 3

El hallazgo en la RMN de una ocupación de uno de los senos frontales por un material hiperintenso homogéneo en secuencias T2, y con densidad de partes blandas y remodelación ósea de las paredes del seno en el TC es altamente sugestivo de:

Familia, que tras una comida campera, acude a urgencias con sialorrea, lagrimeo, vómitos, miosis, disnea e incontinencia de esfínteres. La madre comenta que uno de los niños ha convulsionado, y otro parece confuso y desorientado. En la radiografía de tórax varios de ellos presentan un edema pulmonar, están bradicárdicos, con aumento de la creatinina y de la GOT y la GPT. Usted indicaría:

1. 2. 3. 4. 5.

1. Ingreso en UCI, soporte cardiorrespiratorio y naloxona i.v. 2. Ingreso en UCI, soporte hemodinámico y respiratorio, silibilinina o ácido tióctico y penicilina G sódica.

Mucocele frontal. Osteoma frontal. Papiloma invertido. Adenocarcinoma nasal. Carcinoma epidermoide nasal.

-33-

booksmedicos.org 3. Ingreso en UCI, soporte hemodinámico y respiratorio, oxigenoterapia, fluidoterapia y penicilina procaína. 4. Ingreso en planta, reposo alimenticio, fluidoterapia y carbón activado. 5. Lavado gastrointestinal, control del dolor y dar de alta aconsejando reposo alimenticio.

191.

1. Asumiremos que el paciente es alérgico a algún alérgeno no incluido en la batería de pruebas cutáneas. 2. Los niveles normales de C1inh en sangre nos descartan la posibilidad de un diagnóstico de angioedema hereditario. 3. Lo más probable es que se trate de una intoxicación alimentaria. 4. Deberíamos descartar un angiodema hereditario mediante la determinación de la actividad funcional del C1 inhibidor. 5. No tenemos datos sugestivos de ninguna patología concreta.

¿Cuál de los siguientes fármacos estaría indicado en un caso de lupus eritematoso sistémico con afectación exclusiva cutáneo-articular y en el que es preciso reducir la dosis de esteroides debido a la intensa osteopenia que le ha ocasionado este tratamiento? 194. 1. 2. 3. 4. 5.

192.

Levamisol. Fenilbutazona. Calcitonina. Hidroxicloroquina. D-Penicilamina.

En las anastomosis de esófago, con respecto a la fuga anastomótica, señale la correcta:

1. Es frecuente porque el esófago no tiene muscular. 2. Es lo mismo que una dehiscencia de la anastomosis. 3. Tiene menos riesgo que una anastomosis de intestino delgado. 4. Junto con las resecciones anteriores de recto son las que más riesgo presentan. 5. El tratamiento suele ser, de entrada, quirúrgico.

Paciente de 77 años de edad que consulta por bocio. La paciente aporta una ecografía cervical en la que se informa de aumento de la glándula tiroidea sin lesiones nodulares evidentes y una analítica en la que se incluye una determinación normal de TSH. ¿Cuál sería el diagnóstico y tratamiento oportuno?

195.

1. Bocio por hipertiroidismo primario severo. Precisa tratamiento quirúrgico urgente ya que la TSH es normal. 2. Bocio por hipotiroidismo primario severo. Precisa tratamiento quirúrgico urgente ya que la TSH es normal. 3. Bocio simple. Debemos interrogar a la paciente sobre la presencia de clínica compresiva. 4. Bocio maligno. Debemos remitir a la paciente a cirugía para tiroidectomía total. 5. El aumento del tamaño tiroideo no es una patología en sí misma y no precisa tratamiento.

193.

SIMULACRO 3

Paciente de 15 años de edad, refiere ver peor la pizarra desde hace unos meses. La agudeza visual de lejos es 0,2 en su ojo derecho y 0,4 en su ojo izquierdo. Sin embargo, a través del agujero estenopeico, mejora a 1 en ambos ojos. Lo más probable es que:

1. Simule, de otro modo no es explicable esta mejoría. 2. Presente un glaucoma juvenil. Por ello hay que pedirle un campo visual. 3. Presente un defecto de refracción. Hay que graduarle. 4. Presente una catarata congénita. 5. Presente retinopatía diabética.

Varón de 18 años de edad, que refiere episodios de lesiones cutáneas tipo angioedema en párpados y labios, en ocasiones se acompaña de disnea. No refiere antecedentes de hipersensibilidad alérgica, además de que tras realizarle las pruebas cutáneas a los principales alérgenos, éstas resultan negativas, así como la serología IgE para anisakis. Tras realizarle unas pruebas analíticas, se encuentra con unos niveles de C4 y CH50 bajos con C1inh con niveles normales en sangre. ¿Cuál es la actitud más correcta?

196.

Las pruebas de DNA pueden ser utilizadas para:

1. Estudios médicos forenses para identificar cadáveres. 2. Filiación paterna. 3. Diagnóstico presintomático de enfermedades genéticas. 4. Diagnóstico intraútero de síndromes malformativos. 5. Todas las anteriores.

-34-

booksmedicos.org 197.

El endometrio proliferativo es debido a la acción de:

1. 2. 3. 4. 5.

198.

¿Cuál de los siguientes músculos extiende el antebrazo?

1. 2. 3. 4. 5.

199.

Tríceps. Bíceps braquial. Braquialis. Coracobraquial. Redondo menor.

La molécula inorgánica más abundante del organismo:

1. 2. 3. 4. 5.

200.

Los estrógenos. La progesterona. La prolactina. La ovulación. Andrógenos.

Proteínas. Ácido Nucleicos. Carbohidratos. Agua. Lípidos.

¿Qué zona cardíaca podemos valorar con las derivaciones II, III y aVF del electrocardiograma?

1. 2. 3. 4. 5.

Septo. Posterior. Anterolateral. Inferior. Aurícula derecha.

-35-

SIMULACRO 3

booksmedicos.org

booksmedicos.org

respuestas simulacro 3 Plantilla

NUMERO DE MESA ................................................................................................... VERSION DEL CUESTIONARIO DE EXAMEN ................................................................ Nº DE EXPEDIENTE ...................................................................................................... Nº DE D.N.I. O EQUIVALENTE PARA EXTRANJEROS ................................................... APELLIDOS Y NOMBRE ................................................................................................ ADVERTENCIAS: a) Escriba con BOLIGRAFO sobre superficie dura y lisa. b) No utilice lápiz, rotulador o pluma de tinta que se pueda borrar. c) Indique la respuesta que crea correcta en forma claramente legible. d) Las respuestas ilegibles o confusas o las indicadas con otra clase diferente de signos se penalizarán como incorrectas. e) Las equivocaciones deben subsanarse tachando íntegramente la respuesta errónea, sin que ésta quede visible, y escribiendo al lado la respuesta elegida. f) Si inutilizara esta hoja de respuesta devuelva los tres ejemplares de que se compone a la Mesa de Examen para recibir otra de repuesto y no olvide consignar sus datos personales.

1 2 3 4 5 6 7 8 9 10 11 12 13 14 15 16 17 18 19 20 21 22 23 24 25 26 27 28 29 30 31 32 33 34 35

B E B D B C D B C D C D B D D D B C C A C D C C B B C B E B C C C C A

36 37 38 39 40 41 42 43 44 45 46 47 48 49 50 51 52 53 54 55 56 57 58 59 60 61 62 63 64 65 66 67 68 69 70

D C A D E C B E A C D B A A B E D D D C C E C D C B B C C E A C A A D

71 72 73 74 75 76 77 78 79 80 81 82 83 84 85 86 87 88 89 90 91 92 93 94 95 96 97 98 99 100 101 102 103 104 105

B D D C C D D B C D E C C D D C D C E E E D E D E B D B B E B D C C E

106 107 108 109 110 111 112 113 114 115 116 117 118 119 120 121 122 123 124 125 126 127 128 129 130 131 132 133 134 135 136 137 138 139 140

E C D C D D A A D D E D A D E E E E B C E D B B D C D D B B D B D C B

141 142 143 144 145 146 147 148 149 150 151 152 153 154 155 156 157 158 159 160 161 162 163 164 165 166 167 168 169 170 171 172 173 174 175

C C A E B A B D B A C C C C A C E C D B B E A C D E C C E A A D E C B

176 177 178 179 180 181 182 183 184 185 186 187 188 189 190 191 192 193 194 195 196 197 198 199 200

B E E B B A D B D A D A C E D D C D D C E A A D D

View more...

Comments

Copyright ©2017 KUPDF Inc.
SUPPORT KUPDF